What are wavefunctions?

56 views
Skip to first unread message

Edgar L. Owen

unread,
Dec 27, 2013, 8:19:47 PM12/27/13
to everyth...@googlegroups.com
All,

I'm starting a new topic on wavefunctions in this reply to Jason because he brings up a very important issue.

The usual interpretation of wavefunctions are that particles are 'spread out' in the fixed common pre-existing space that quantum theory mistakenly assumes, that they are superpostions of states in this space.

However in my book on Reality in Part III, Elementals I propose another interpretation, namely that particles are discrete information entities in logical computational space, and that what wavefunctions actually are is descriptions of how space can become dimensionalized by decoherence events (since decoherence events produce exact conserved relationships between the dimensional variables of interacting particles). The mathematical results are exactly the same, its just a different interpretation.

However this approach that space is something that emerges from quantum events rather than being a fixed pre-existing background to events enables us to conceptually unify GR and QM and also resolves all so called quantum 'paradox' as quantum processes are paradoxical ONLY with respect to the fixed pre-existing space mistakenly assumed.

Edgar




Jason Resch

unread,
Dec 27, 2013, 9:17:52 PM12/27/13
to Everything List
On Fri, Dec 27, 2013 at 8:19 PM, Edgar L. Owen <edga...@att.net> wrote:
All,

I'm starting a new topic on wavefunctions in this reply to Jason because he brings up a very important issue.

The usual interpretation of wavefunctions are that particles are 'spread out' in the fixed common pre-existing space that quantum theory mistakenly assumes, that they are superpostions of states in this space.

However in my book on Reality in Part III, Elementals I propose another interpretation, namely that particles are discrete information entities in logical computational space, and that what wavefunctions actually are is descriptions of how space can become dimensionalized by decoherence events (since decoherence events produce exact conserved relationships between the dimensional variables of interacting particles).

I am not sure that I follow, but it sounds like an interesting idea. It reminds me of Ron Garret's talk, where he says metaphorically "we live in a simulation running on a quantum computer": http://www.youtube.com/watch?v=dEaecUuEqfc
 
The mathematical results are exactly the same, its just a different interpretation.

I am not sure if it is possible in any theory consistent with QM to deny completely the notion of superposition. How can the single-electron double-slit experiment be explained without the electron being in more than one place at the same time?

I think it would help me understand your interpretation if you answered the following questions. According to your interpretation:

1. Are faster-than-light influences involved?
2. When it is determined whether or not Schrodinger's cat is alive or dead?
3. Are quantum computers possible, and if so, where are all the intermediate computations performed?

Jason

LizR

unread,
Dec 27, 2013, 9:23:07 PM12/27/13
to everyth...@googlegroups.com
There is certainly evidence that "particles" are small amounts of digital information. Garrett Lisi's ESTOE http://en.wikipedia.org/wiki/An_Exceptionally_Simple_Theory_of_Everything for example assumes this, and it is part of the support for mathematical theories of reality like Tegmark's (imho).

LizR

unread,
Dec 27, 2013, 9:27:17 PM12/27/13
to everyth...@googlegroups.com
On 28 December 2013 14:19, Edgar L. Owen <edga...@att.net> wrote:

enables us to conceptually unify GR and QM and also resolves all so called quantum 'paradox' as quantum processes are paradoxical ONLY with respect to the fixed pre-existing space mistakenly assumed.

I would expect any attempt at a TOE to at least do the above. It should also, of course, make unexpected and testable predictions.

Edgar L. Owen

unread,
Dec 27, 2013, 10:08:15 PM12/27/13
to everyth...@googlegroups.com
Jason,

Answers to your 3 questions. 

1. No.
2. Determined by which observer? The cat is always either dead or alive. It's just a matter of someone making a measurement to find out.
3. Of course quantum computers are possible. Simple examples already exist, but fundamentally all computations take place in logical information space, as I've described before in a number of posts.

However I don't think the answers to these questions will help you understand the theory. Refer to my other topic on this group titled "Yes, my book does cover quantum reality", or refer to the book itself, or I can explain further....


Edgar

Jason Resch

unread,
Dec 27, 2013, 10:26:07 PM12/27/13
to Everything List
On Fri, Dec 27, 2013 at 10:08 PM, Edgar L. Owen <edga...@att.net> wrote:
Jason,

Answers to your 3 questions. 

1. No.

If there are no faster-than-light (FTL) influences, then how does your interpretation address the EPR paradox ( http://en.wikipedia.org/wiki/EPR_paradox )?  As a previously mentioned, according to Bell's theorem, there is only one known solution to the paradox that does not involve FTL influences, and that is Everett's theory of many-worlds.
 
2. Determined by which observer? The cat is always either dead or alive. It's just a matter of someone making a measurement to find out.

So are you saying that before the measurement the cat is neither alive nor dead, both alive and dead, or definitely alive or definitely dead?  If you, (and I think you are), saying that the cat is always definitely alive or definitely dead, then about about the radioactive atom? Is it ever in a state of being decayed and not decayed? If you say no, it sounds like you are denying the reality of the superposition, which some interpretations do, but then this leads to difficulties explaining how quantum computers work (which require the superposition to exist).
 
3. Of course quantum computers are possible. Simple examples already exist, but fundamentally all computations take place in logical information space, as I've described before in a number of posts.

If a quantum computer can factor a randomly generated semi-prime of 1,000,000 digits, where is the computation for this being performed? This is a computation that is so complex that no conventional computer (even the size of the universe) could solve this problem if given a trillion years, yet a device that could fit on your desk could solve it in less than a second. If the exponentially exploding states in the superposition are not really there, there is apparently no explanation at all for where the result of the computation comes from.
 

However I don't think the answers to these questions will help you understand the theory. Refer to my other topic on this group titled "Yes, my book does cover quantum reality", or refer to the book itself, or I can explain further....


Thanks. I may not have time to read your book for some time, so for now I would prefer to proceed by e-mail, at least until some resolution is reached. I appreciate the time you have spent so far in answering my questions.

Jason
 



On Friday, December 27, 2013 9:17:52 PM UTC-5, Jason wrote:



On Fri, Dec 27, 2013 at 8:19 PM, Edgar L. Owen <edga...@att.net> wrote:
All,

I'm starting a new topic on wavefunctions in this reply to Jason because he brings up a very important issue.

The usual interpretation of wavefunctions are that particles are 'spread out' in the fixed common pre-existing space that quantum theory mistakenly assumes, that they are superpostions of states in this space.

However in my book on Reality in Part III, Elementals I propose another interpretation, namely that particles are discrete information entities in logical computational space, and that what wavefunctions actually are is descriptions of how space can become dimensionalized by decoherence events (since decoherence events produce exact conserved relationships between the dimensional variables of interacting particles).

I am not sure that I follow, but it sounds like an interesting idea. It reminds me of Ron Garret's talk, where he says metaphorically "we live in a simulation running on a quantum computer": http://www.youtube.com/watch?v=dEaecUuEqfc
 
The mathematical results are exactly the same, its just a different interpretation.

I am not sure if it is possible in any theory consistent with QM to deny completely the notion of superposition. How can the single-electron double-slit experiment be explained without the electron being in more than one place at the same time?

I think it would help me understand your interpretation if you answered the following questions. According to your interpretation:

1. Are faster-than-light influences involved?
2. When it is determined whether or not Schrodinger's cat is alive or dead?
3. Are quantum computers possible, and if so, where are all the intermediate computations performed?

Jason
 

However this approach that space is something that emerges from quantum events rather than being a fixed pre-existing background to events enables us to conceptually unify GR and QM and also resolves all so called quantum 'paradox' as quantum processes are paradoxical ONLY with respect to the fixed pre-existing space mistakenly assumed.


 

--
You received this message because you are subscribed to the Google Groups "Everything List" group.
To unsubscribe from this group and stop receiving emails from it, send an email to everything-li...@googlegroups.com.
To post to this group, send email to everyth...@googlegroups.com.
Visit this group at http://groups.google.com/group/everything-list.
For more options, visit https://groups.google.com/groups/opt_out.

Edgar L. Owen

unread,
Dec 27, 2013, 10:27:58 PM12/27/13
to everyth...@googlegroups.com
Jason,

PS to answer your other question. In the double slit experiment there is no pre-existing dimensional space for the electron to be in more than one place in. Everything is being computed exactly in the fundamental non-physical dimensionless information space. What we call space is actually networks of dimensional relationships between quantum events that emerge from those quantum events. Empty space is unobservable and therefore not a part of reality. All that is observable is events, in this case specifically the dimensional relationships between the participants in quantum events imposed by the conservation laws. But his occurs in logical (non-dimensional) computational space, not a physical dimensional space.

So in the double slit experiment the actual events are the decoherences of the electrons with the screen which produce exact dimensional relationships. The apparent wave behavior of the electrons passing through the slits is a non-observable backward inference based on the wavefunction equations which are not electrons spread out in multiple locations in a pre-existing space but the mathematical equivalent probabilities of how space could dimensionalize when those electrons decohere.

This is a subtle theory, and hopefully I can explain further if necessary, or you can read Part III of my book.....

Edgar


On Friday, December 27, 2013 9:17:52 PM UTC-5, Jason wrote:

LizR

unread,
Dec 27, 2013, 10:28:45 PM12/27/13
to everyth...@googlegroups.com
On 28 December 2013 16:26, Jason Resch <jason...@gmail.com> wrote:



On Fri, Dec 27, 2013 at 10:08 PM, Edgar L. Owen <edga...@att.net> wrote:
Jason,

Answers to your 3 questions. 

1. No.

If there are no faster-than-light (FTL) influences, then how does your interpretation address the EPR paradox ( http://en.wikipedia.org/wiki/EPR_paradox )?  As a previously mentioned, according to Bell's theorem, there is only one known solution to the paradox that does not involve FTL influences, and that is Everett's theory of many-worlds.

Huw Price's time symmetry also solves the paradox. Bell agreed with him on this, so I think it's probably a valid result even if not widely known. I'm not sure that Price's ontology is intended as a "rival" to Everett, however, although it may introduce modifications.

Edgar L. Owen

unread,
Dec 27, 2013, 10:33:30 PM12/27/13
to everyth...@googlegroups.com
Liz,

What I haven't deciphered in Lisi's theory is what its elementals are. He seems to have come up with a set of elemental particle properties that populate his E8 group exactly and completely but they do not all appear to be commonly recognized particle properties such as charges, spins, etc. 

Can anyone give me a list of what Lisi's group elements actually correspond to in particle physics?

Edgar

Edgar L. Owen

unread,
Dec 27, 2013, 10:58:29 PM12/27/13
to everyth...@googlegroups.com
Jason,

All your questions assume a pre-existing space that doesn't actually exist. When it is recognized that space emerges from events rather than being a fixed background to them these questions disappear.

E.g. in the EPR 'paradox' the opposite spin relationship of the two particles is fixed when they are created by the particle property conservation law, but the absolutely crucial point is that that when it is created that relationship is only in the mutual frame of the two particles which is not yet connected to the frame of the observer. It is only when the frame of the particles and the observer are aligned by a common dimensional event (the measurement of the spin of one particle by the observer) that both frames become aligned and thus the spin of the second particle becomes apparent in the observer's frame.

The exact spin relationship between the particles existed since their creation. It had to since their creation determined it. However that frame was independent of that of the observer until a single common event connected the two frames at which time every dimensional relationship of both frames became aligned. It is basically how two independent spaces must be completely ignorant of each other until connected by a common dimensional event at which point all dimensionality of both become automatically aligned in a single dimensionality.

Thus there is NO need for faster than light transmission, and your "As a previously mentioned, according to Bell's theorem, there is only one known solution to the paradox that does not involve FTL influences, and that is Everett's theory of many-worlds." is certainly not true (more accurately does not apply) in this model.


Second, the cat is always either alive or dead in its own frame. But that frame is unknowable by some external observer until it becomes observable via a common event between that frame and that observer's frame (the measurement of whether it is alive or dead).

We can't assume some single universal dimensional frame. All dimensional frames arise independently of each other and unaligned with each other (because there is no common fixed pre-existing standard frame of reference, there are only individual independent frames emerging from connected networks of dimensional events) until they are connected and then dimensionally aligned by some shared event.

Edgar

Craig Weinberg

unread,
Dec 27, 2013, 11:43:19 PM12/27/13
to everyth...@googlegroups.com
I also suspect that quantum makes spacetime rather than being phenomena which take place in spacetime, if that's what you're proposing. I'm not sure however that explaining physical space as information space is ultimately an improvement. Without linking either one to awareness, the result is still that we are explaining a universe that we can never experience.

Craig

LizR

unread,
Dec 27, 2013, 11:52:51 PM12/27/13
to everyth...@googlegroups.com
Most TOEs try to get space-time as emergent from something simpler.


Edgar L. Owen

unread,
Dec 27, 2013, 11:53:48 PM12/27/13
to everyth...@googlegroups.com
Craig,

Yes, I'm proposing that spacetime emerges from quantum events. But your second question depends on this since if spacetime emerges from quantum events there can be no physical space since physical space is exactly what we agreed doesn't exist until it emerges from quantum events which are information space computations.

And of course we can experience the universe, though only through our own filters. If we could have no knowledge of reality we could not function within it and could not exist. 

And our own mental world view is part of reality and we have direct knowledge of that part of reality...

Edgar

Jason Resch

unread,
Dec 28, 2013, 12:28:55 AM12/28/13
to Everything List
On Fri, Dec 27, 2013 at 10:27 PM, Edgar L. Owen <edga...@att.net> wrote:
Jason,

PS to answer your other question. In the double slit experiment there is no pre-existing dimensional space for the electron to be in more than one place in.

Then what is it interfering with if not itself?
 
Everything is being computed exactly in the fundamental non-physical dimensionless information space. What we call space is actually networks of dimensional relationships between quantum events that emerge from those quantum events. Empty space is unobservable and therefore not a part of reality.

It has observable effects, such as the amount of delay it can introduce between the emission and reception of a photon that must travel through it.
 
All that is observable is events, in this case specifically the dimensional relationships between the participants in quantum events imposed by the conservation laws. But his occurs in logical (non-dimensional) computational space, not a physical dimensional space.

This is beginning to sound a lot like the UD.
 

So in the double slit experiment the actual events are the decoherences of the electrons with the screen which produce exact dimensional relationships. The apparent wave behavior of the electrons passing through the slits is a non-observable backward inference based on the wavefunction equations which are not electrons spread out in multiple locations in a pre-existing space but the mathematical equivalent probabilities of how space could dimensionalize when those electrons decohere.

This sounds a bit like the pilot-wave theory.
 

This is a subtle theory, and hopefully I can explain further if necessary, or you can read Part III of my book.....


There is a comparison table of various interpretations of QM.  I am curious how yours would appear if it were added to it:

Interpretation Author(s)Deterministic? Wavefunction
real?
Unique
history?
Hidden
variables
?
Collapsing
wavefunctions?
Observer
role?
Local? Counterfactual
definiteness
?
Universal
wavefunction

exists?
Ensemble interpretation Max Born, 1926 AgnosticNo YesAgnosticNo NoNoNo No
Copenhagen interpretation Niels BohrWerner Heisenberg, 1927 NoNo1 YesNo Yes2 CausalNo NoNo
de Broglie–Bohm theory Louis de Broglie, 1927,David Bohm, 1952 YesYes3 Yes4 YesNo NoNoYes Yes
von Neumann interpretation John von Neumann, 1932, John Archibald WheelerEugene Wigner NoYes YesNoYes CausalNoNo Yes
Quantum logic Garrett Birkhoff, 1936 AgnosticAgnostic Yes5 NoNo Interpretational6 AgnosticNo No
Many-worlds interpretation Hugh Everett, 1957 YesYes NoNoNo NoYesNo Yes
Popper's interpretation[51] Karl Popper, 1957[52] NoYes YesYesNo NoYesYes13 No
Time-symmetric theories Satosi Watanabe, 1955 YesYes YesYesNo NoYesNo Yes
Stochastic interpretation Edward Nelson, 1966 NoNo YesNoNo NoNoNo No
Many-minds interpretation H. Dieter Zeh, 1970 YesYes NoNoNo Interpretational7 YesNo Yes
Consistent histories Robert B. Griffiths, 1984 Agnostic8 Agnostic8 NoNo NoInterpretational6 YesNo No
Objective collapse theories Ghirardi–Rimini–Weber, 1986,
Penrose interpretation, 1989
NoYesYes NoYesNo NoNoNo
Transactional interpretation John G. Cramer, 1986 NoYesYes NoYes9 NoNo14 YesNo
Relational interpretation Carlo Rovelli, 1994 AgnosticNoAgnostic10 NoYes11 Intrinsic12 YesNo No

(My appologies for anyone's e-mail program that has difficulty parsing the above table, it is taken from: http://en.wikipedia.org/wiki/Interpretations_of_quantum_mechanics )

Jason

 
Edgar


On Friday, December 27, 2013 9:17:52 PM UTC-5, Jason wrote:



On Fri, Dec 27, 2013 at 8:19 PM, Edgar L. Owen <edga...@att.net> wrote:
All,

I'm starting a new topic on wavefunctions in this reply to Jason because he brings up a very important issue.

The usual interpretation of wavefunctions are that particles are 'spread out' in the fixed common pre-existing space that quantum theory mistakenly assumes, that they are superpostions of states in this space.

However in my book on Reality in Part III, Elementals I propose another interpretation, namely that particles are discrete information entities in logical computational space, and that what wavefunctions actually are is descriptions of how space can become dimensionalized by decoherence events (since decoherence events produce exact conserved relationships between the dimensional variables of interacting particles).

I am not sure that I follow, but it sounds like an interesting idea. It reminds me of Ron Garret's talk, where he says metaphorically "we live in a simulation running on a quantum computer": http://www.youtube.com/watch?v=dEaecUuEqfc
 
The mathematical results are exactly the same, its just a different interpretation.

I am not sure if it is possible in any theory consistent with QM to deny completely the notion of superposition. How can the single-electron double-slit experiment be explained without the electron being in more than one place at the same time?

I think it would help me understand your interpretation if you answered the following questions. According to your interpretation:

1. Are faster-than-light influences involved?
2. When it is determined whether or not Schrodinger's cat is alive or dead?
3. Are quantum computers possible, and if so, where are all the intermediate computations performed?

Jason
 

However this approach that space is something that emerges from quantum events rather than being a fixed pre-existing background to events enables us to conceptually unify GR and QM and also resolves all so called quantum 'paradox' as quantum processes are paradoxical ONLY with respect to the fixed pre-existing space mistakenly assumed.


 

--

Jason Resch

unread,
Dec 28, 2013, 12:39:44 AM12/28/13
to Everything List
On Fri, Dec 27, 2013 at 10:28 PM, LizR <liz...@gmail.com> wrote:
On 28 December 2013 16:26, Jason Resch <jason...@gmail.com> wrote:



On Fri, Dec 27, 2013 at 10:08 PM, Edgar L. Owen <edga...@att.net> wrote:
Jason,

Answers to your 3 questions. 

1. No.

If there are no faster-than-light (FTL) influences, then how does your interpretation address the EPR paradox ( http://en.wikipedia.org/wiki/EPR_paradox )?  As a previously mentioned, according to Bell's theorem, there is only one known solution to the paradox that does not involve FTL influences, and that is Everett's theory of many-worlds.

Huw Price's time symmetry also solves the paradox.

Is this the same as, or related to Cramer's transactional interpretation?
 
Bell agreed with him on this, so I think it's probably a valid result even if not widely known. I'm not sure that Price's ontology is intended as a "rival" to Everett, however, although it may introduce modifications.

Interesting, do you have any sources you can point me to on this?

Thanks,

Jason

meekerdb

unread,
Dec 28, 2013, 12:43:44 AM12/28/13
to everyth...@googlegroups.com
On 12/27/2013 7:26 PM, Jason Resch wrote:



On Fri, Dec 27, 2013 at 10:08 PM, Edgar L. Owen <edga...@att.net> wrote:
Jason,

Answers to your 3 questions. 

1. No.

If there are no faster-than-light (FTL) influences, then how does your interpretation address the EPR paradox ( http://en.wikipedia.org/wiki/EPR_paradox )?  As a previously mentioned, according to Bell's theorem, there is only one known solution to the paradox that does not involve FTL influences, and that is Everett's theory of many-worlds.

That's not really true.  If you look at the wikipedia table that you cited, http://en.wikipedia.org/wiki/Interpretations_of_quantum_mechanics  you see that Popper's, time symmetric, many-minds, consistent histories, and the relational interpretation are all local, i.e. no FTL.  To that I would add the purely epistemic "non-intepretation" of Peres and Fuchs.


 
2. Determined by which observer? The cat is always either dead or alive. It's just a matter of someone making a measurement to find out.

So are you saying that before the measurement the cat is neither alive nor dead, both alive and dead, or definitely alive or definitely dead?  If you, (and I think you are), saying that the cat is always definitely alive or definitely dead, then about about the radioactive atom? Is it ever in a state of being decayed and not decayed? If you say no, it sounds like you are denying the reality of the superposition, which some interpretations do, but then this leads to difficulties explaining how quantum computers work (which require the superposition to exist).

Superposition is just a question of basis.  An eigenstate in one basis is a superposition in another.


 
3. Of course quantum computers are possible. Simple examples already exist, but fundamentally all computations take place in logical information space, as I've described before in a number of posts.

If a quantum computer can factor a randomly generated semi-prime of 1,000,000 digits, where is the computation for this being performed? This is a computation that is so complex that no conventional computer (even the size of the universe) could solve this problem if given a trillion years, yet a device that could fit on your desk could solve it in less than a second. If the exponentially exploding states in the superposition are not really there, there is apparently no explanation at all for where the result of the computation comes from.

However, keep in mind that all this computation takes place in this world, otherwise the processes could not interfere and converge to a (probable) answer. 

Brent

Jason Resch

unread,
Dec 28, 2013, 12:51:50 AM12/28/13
to Everything List
On Fri, Dec 27, 2013 at 10:58 PM, Edgar L. Owen <edga...@att.net> wrote:
Jason,

All your questions assume a pre-existing space that doesn't actually exist. When it is recognized that space emerges from events rather than being a fixed background to them these questions disappear.

If the appearance of space is emergent, then shouldn't the appearance of time be as well?
 

E.g. in the EPR 'paradox' the opposite spin relationship of the two particles is fixed when they are created by the particle property conservation law, but the absolutely crucial point is that that when it is created that relationship is only in the mutual frame of the two particles which is not yet connected to the frame of the observer. It is only when the frame of the particles and the observer are aligned by a common dimensional event (the measurement of the spin of one particle by the observer) that both frames become aligned and thus the spin of the second particle becomes apparent in the observer's frame.

Yes, the original EPR paper is what motivated Einstein, Podolsky, and Rosen to propose there were hidden variables (which is what you propose above in saying the "spin relation of the two particles is fixed when they are created"). However, under Bell's modification to the EPR case, he found that supposing such hidden variables have a single definite state prior to measurement is impossible and cannot work.  This becomes evident when you measure something such as the polarization of photons at angles other than 0, 45, or 90 degrees, where the agreements are 100%, 50%, and 0%.  If instead, you measure at angles like 30%, you find the agreement is 75%, which is higher than is mathematically possible assuming the photons have "single, pre-determined properties" prior to the measurement.
 

The exact spin relationship between the particles existed since their creation. It had to since their creation determined it. However that frame was independent of that of the observer until a single common event connected the two frames at which time every dimensional relationship of both frames became aligned. It is basically how two independent spaces must be completely ignorant of each other until connected by a common dimensional event at which point all dimensionality of both become automatically aligned in a single dimensionality.

The only way the particles can have their properties determined at the time of creation, and remain compatible with Bell's theorem, is if the properties of the particles are in a mult-valued (superposed) state.
 

Thus there is NO need for faster than light transmission, and your "As a previously mentioned, according to Bell's theorem, there is only one known solution to the paradox that does not involve FTL influences, and that is Everett's theory of many-worlds." is certainly not true (more accurately does not apply) in this model.

This is exactly the case Bell's theorem applies to, the notion of single definite values prior to measurement. This is not clear from reading only about the EPR paradox, you need to read through Bell's paper (or the website I provided that gave a walk through of it).

meekerdb

unread,
Dec 28, 2013, 1:20:03 AM12/28/13
to everyth...@googlegroups.com
On 12/27/2013 7:58 PM, Edgar L. Owen wrote:
Jason,

All your questions assume a pre-existing space that doesn't actually exist. When it is recognized that space emerges from events rather than being a fixed background to them these questions disappear.

E.g. in the EPR 'paradox' the opposite spin relationship of the two particles is fixed when they are created by the particle property conservation law, but the absolutely crucial point is that that when it is created that relationship is only in the mutual frame of the two particles which is not yet connected to the frame of the observer. It is only when the frame of the particles and the observer are aligned by a common dimensional event (the measurement of the spin of one particle by the observer) that both frames become aligned and thus the spin of the second particle becomes apparent in the observer's frame.

The problem is that "when" and "become" refer to a time dimension and, when the measurements are spacelike, there is no canonical ordering to the measurement events.



The exact spin relationship between the particles existed since their creation.

That's a hidden variable which violation of Bell's inequality rules out unless the relationship is spacelike (i.e. FTL).


It had to since their creation determined it. However that frame was independent of that of the observer until a single common event connected the two frames at which time every dimensional relationship of both frames became aligned. It is basically how two independent spaces must be completely ignorant of each other until connected by a common dimensional event at which point all dimensionality of both become automatically aligned in a single dimensionality.

Thus there is NO need for faster than light transmission, and your "As a previously mentioned, according to Bell's theorem, there is only one known solution to the paradox that does not involve FTL influences, and that is Everett's theory of many-worlds." is certainly not true (more accurately does not apply) in this model.


Second, the cat is always either alive or dead in its own frame. But that frame is unknowable by some external observer until it becomes observable via a common event between that frame and that observer's frame (the measurement of whether it is alive or dead).

We can't assume some single universal dimensional frame. All dimensional frames arise independently of each other and unaligned with each other (because there is no common fixed pre-existing standard frame of reference, there are only individual independent frames emerging from connected networks of dimensional events) until they are connected and then dimensionally aligned by some shared event.

So there's a global time coordinate, but no global space coordinates?

Brent

LizR

unread,
Dec 28, 2013, 1:26:27 AM12/28/13
to everyth...@googlegroups.com
On 28 December 2013 18:39, Jason Resch <jason...@gmail.com> wrote:
On Fri, Dec 27, 2013 at 10:28 PM, LizR <liz...@gmail.com> wrote:
On 28 December 2013 16:26, Jason Resch <jason...@gmail.com> wrote:
On Fri, Dec 27, 2013 at 10:08 PM, Edgar L. Owen <edga...@att.net> wrote:
Jason,

Answers to your 3 questions. 

1. No.

If there are no faster-than-light (FTL) influences, then how does your interpretation address the EPR paradox ( http://en.wikipedia.org/wiki/EPR_paradox )?  As a previously mentioned, according to Bell's theorem, there is only one known solution to the paradox that does not involve FTL influences, and that is Everett's theory of many-worlds.

Huw Price's time symmetry also solves the paradox.

Is this the same as, or related to Cramer's transactional interpretation?

No, it's a lot simpler. It doesn't add any new physics, and removes one assumption. 
 
Bell agreed with him on this, so I think it's probably a valid result even if not widely known. I'm not sure that Price's ontology is intended as a "rival" to Everett, however, although it may introduce modifications.

Interesting, do you have any sources you can point me to on this?

I'd start with "Time's arrow and Archimedes' point" by Huw Price.

Jason Resch

unread,
Dec 28, 2013, 1:31:22 AM12/28/13
to Everything List
On Sat, Dec 28, 2013 at 12:43 AM, meekerdb <meek...@verizon.net> wrote:
On 12/27/2013 7:26 PM, Jason Resch wrote:



On Fri, Dec 27, 2013 at 10:08 PM, Edgar L. Owen <edga...@att.net> wrote:
Jason,

Answers to your 3 questions. 

1. No.

If there are no faster-than-light (FTL) influences, then how does your interpretation address the EPR paradox ( http://en.wikipedia.org/wiki/EPR_paradox )?  As a previously mentioned, according to Bell's theorem, there is only one known solution to the paradox that does not involve FTL influences, and that is Everett's theory of many-worlds.

That's not really true.  If you look at the wikipedia table that you cited, http://en.wikipedia.org/wiki/Interpretations_of_quantum_mechanics  you see that Popper's

Popper's interpretation uses hidden variables, and hence is ruled out by Bell's theorem. It also predicts that FTL signaling of information should be possible.
 
, time symmetric,

I couldn't find much information about Satosi Watanabe's theory, from the table it also appears to have hidden variables.  Also, if it is both deterministic, and the universal wave function is real, I don't see how under QM this can lead to a unique history. (as the table indicates).  Do you understand how this can be?
 
many-minds,

Zeh's many-minds, like many-worlds, suggests there is no unique history.  E.g., there are multiple outcomes for each measurement.
 
consistent histories,

James Hartle, who wrote a book about consistent histories with Gell-Mann, said that consistent histories is the same as many worlds ( http://onqm.blogspot.com/2009_08_01_archive.html ).
 
and the relational interpretation

The relational interpretation is "agnostic" on whether or not there is a unique history.  Is this many-worlds in denial?  It seems to allow one observer to be in a superposition, relative to another. This is an example of an observer in more than one state at a time, which of  course leads to multiple-outcome measurements.
 
are all local, i.e. no FTL. 

Yes, but it seems all local (and valid) interpretations of QM allow measurements to have more than one outcome.
 
To that I would add the purely epistemic "non-intepretation" of Peres and Fuchs.
 
"No interpretation needed" -- I can interpret this in two ways, one way is to just take the math and equations literally (this leads to Everett), the other is "shut up and calculate", which leads no where really.


 

 
2. Determined by which observer? The cat is always either dead or alive. It's just a matter of someone making a measurement to find out.

So are you saying that before the measurement the cat is neither alive nor dead, both alive and dead, or definitely alive or definitely dead?  If you, (and I think you are), saying that the cat is always definitely alive or definitely dead, then about about the radioactive atom? Is it ever in a state of being decayed and not decayed? If you say no, it sounds like you are denying the reality of the superposition, which some interpretations do, but then this leads to difficulties explaining how quantum computers work (which require the superposition to exist).

Superposition is just a question of basis.  An eigenstate in one basis is a superposition in another.


Can you provide a concrete example where some system can simultaneously be considered to be both in a superposition and not?  Is this like the superposition having collapsed for Wigner's friend while remaining for Wigner before he enters the room?
 

 
3. Of course quantum computers are possible. Simple examples already exist, but fundamentally all computations take place in logical information space, as I've described before in a number of posts.

If a quantum computer can factor a randomly generated semi-prime of 1,000,000 digits, where is the computation for this being performed? This is a computation that is so complex that no conventional computer (even the size of the universe) could solve this problem if given a trillion years, yet a device that could fit on your desk could solve it in less than a second. If the exponentially exploding states in the superposition are not really there, there is apparently no explanation at all for where the result of the computation comes from.

However, keep in mind that all this computation takes place in this world, otherwise the processes could not interfere and converge to a (probable) answer. 

The computation is occurring in a (temporarily) causally isolated system.  It is debatable whether we can rightfully say it happens in any particular world.

Jason

Jason Resch

unread,
Dec 28, 2013, 1:37:57 AM12/28/13
to Everything List
On Sat, Dec 28, 2013 at 1:26 AM, LizR <liz...@gmail.com> wrote:
On 28 December 2013 18:39, Jason Resch <jason...@gmail.com> wrote:
On Fri, Dec 27, 2013 at 10:28 PM, LizR <liz...@gmail.com> wrote:
On 28 December 2013 16:26, Jason Resch <jason...@gmail.com> wrote:
On Fri, Dec 27, 2013 at 10:08 PM, Edgar L. Owen <edga...@att.net> wrote:
Jason,

Answers to your 3 questions. 

1. No.

If there are no faster-than-light (FTL) influences, then how does your interpretation address the EPR paradox ( http://en.wikipedia.org/wiki/EPR_paradox )?  As a previously mentioned, according to Bell's theorem, there is only one known solution to the paradox that does not involve FTL influences, and that is Everett's theory of many-worlds.

Huw Price's time symmetry also solves the paradox.

Is this the same as, or related to Cramer's transactional interpretation?

No, it's a lot simpler. It doesn't add any new physics, and removes one assumption. 

What is that assumption that is removed?
 
 
Bell agreed with him on this, so I think it's probably a valid result even if not widely known. I'm not sure that Price's ontology is intended as a "rival" to Everett, however, although it may introduce modifications.

Interesting, do you have any sources you can point me to on this?

I'd start with "Time's arrow and Archimedes' point" by Huw Price.

Thanks.

Jason

LizR

unread,
Dec 28, 2013, 1:54:50 AM12/28/13
to everyth...@googlegroups.com
On 28 December 2013 19:37, Jason Resch <jason...@gmail.com> wrote:
On Sat, Dec 28, 2013 at 1:26 AM, LizR <liz...@gmail.com> wrote:
On 28 December 2013 18:39, Jason Resch <jason...@gmail.com> wrote:
On Fri, Dec 27, 2013 at 10:28 PM, LizR <liz...@gmail.com> wrote:
On 28 December 2013 16:26, Jason Resch <jason...@gmail.com> wrote:
On Fri, Dec 27, 2013 at 10:08 PM, Edgar L. Owen <edga...@att.net> wrote:
Jason,

Answers to your 3 questions. 

1. No.

If there are no faster-than-light (FTL) influences, then how does your interpretation address the EPR paradox ( http://en.wikipedia.org/wiki/EPR_paradox )?  As a previously mentioned, according to Bell's theorem, there is only one known solution to the paradox that does not involve FTL influences, and that is Everett's theory of many-worlds.

Huw Price's time symmetry also solves the paradox.

Is this the same as, or related to Cramer's transactional interpretation?

No, it's a lot simpler. It doesn't add any new physics, and removes one assumption. 

What is that assumption that is removed?

That simple quantum events have a built in arrow of time. This assumption isn't in the physics, but it's usually in the minds of people when they try to explain EPR, for example, by saying that certain things can't happen "without FTL signalling". Saying this assumes that the particles involved are constrained by what happened to them in the past, but not constrained by what will happen to them in the future. This is a very powerful assumption, built into our nature as macroscopic creatures who are (unfortunately) all too susceptible to the effects of the entropy gradient - but there is no reason it should apply to, for example, individual photons. Assuming that photons act like people as far as the arrow of time goes skews our ideas of what is "reasonable behaviour" for quantum systems, and (according to Prof Price and others) leads us to see lots of things as weird / spooky when they are actually merely exhibiting the time symmetry inherent in the laws of physics.

If we allow past and future constraints to affect particles, for example, any need for FTL effects to explain EPR vanishes, because all the information involved is carried by the particles themselves, which of course never travel FTL. It just happens to be carried in both time directions, with the photon's state in mid-flight affected by both the event that generated it in the past and the measurement that will be applied to it in the future.

Bruno Marchal

unread,
Dec 28, 2013, 6:30:26 AM12/28/13
to everyth...@googlegroups.com
On 28 Dec 2013, at 04:08, Edgar L. Owen wrote:

Jason,

Answers to your 3 questions. 

1. No.
2. Determined by which observer? The cat is always either dead or alive. It's just a matter of someone making a measurement to find out.

Then there is a collapse of the wave. I thought you disagree with collapse. 
Without collapse, if you look at a cat in the superposition dead+alive, you end up yourself being described by a superposition seeing a cat dead + seeing a cat alive. It is equivalent 'computationally) with a self-duplication.

Bruno




--
You received this message because you are subscribed to the Google Groups "Everything List" group.
To unsubscribe from this group and stop receiving emails from it, send an email to everything-li...@googlegroups.com.
To post to this group, send email to everyth...@googlegroups.com.
Visit this group at http://groups.google.com/group/everything-list.
For more options, visit https://groups.google.com/groups/opt_out.

Edgar L. Owen

unread,
Dec 28, 2013, 7:54:56 AM12/28/13
to everyth...@googlegroups.com
Jason,

Clock time is emergent from comp but comp takes place sequentially in P-time, which is effectively the processor cycles of comp. This is another way the clock time P-time distinction works to produce reality as it exists....

No, the particles MUST have their properties determined at the time of creation to obey the law of conservation of particle properties. That is particle physics 101. Specifically in this case their spins must be created equal and opposite but this is only known to their frame, not to that of the observer until he links and aligns it with a measurement.

Edgar

Edgar L. Owen

unread,
Dec 28, 2013, 7:58:42 AM12/28/13
to everyth...@googlegroups.com
Brent,

No, the oppositely aligned spins is NOT a hidden variable and there is no FTL. Reread my post....

Edgar

Jason Resch

unread,
Dec 28, 2013, 10:20:08 AM12/28/13
to everyth...@googlegroups.com


On Dec 28, 2013, at 6:54 AM, "Edgar L. Owen" <edga...@att.net> wrote:

Jason,

Clock time is emergent from comp but comp takes place sequentially in P-time, which is effectively the processor cycles of comp. This is another way the clock time P-time distinction works to produce reality as it exists....

No, the particles MUST have their properties determined at the time of creation to obey the law of conservation of particle properties.

Many worlds says that after the decay the electron is both spin up and spin down and the same for the positron.

However they are correlated such that the super position is:

(e up and p down) + (e down and p up)

When you measure one, you too become part of that superposed state. 

(measured e up and e up and p down) + (measured e down and e down and p up) 

According to many worlds, you end up entangled (correlated with) both of them.  According to collapse theories, only one of them (the other state mysteriously vanishes).

Your proposal that only one definite outcome is set at the time of the pair's creation, that is, the electron is either definitely,and only up or definitely and only down, prior to measurement, is unworkable, as it leads to statistics that are incompatible with observed and predicted quantum mechanics, as Bell showed.

You can't just deny Bell's result or say it doesn't apply to your theory. it was meant to cover exactly the case as you described it.

You should not feel bad that you missed it. Einstein missed it too.

Jason

r />

spudb...@aol.com

unread,
Dec 28, 2013, 11:11:19 AM12/28/13
to everyth...@googlegroups.com
One interpretation by some physicists with Cramer's transactional model, implies that information is coming from the future, and handshaking with the paste to create the present. Price's old book seems to imply this as well.

Edgar L. Owen

unread,
Dec 28, 2013, 1:26:40 PM12/28/13
to everyth...@googlegroups.com
Jason,

No, you simply don't understand what I'm saying, what my model is. There are two independent separate mini spacetime fragments here. When you understand that you will see how it works and avoids the problems you point out...

You should not feel bad that you missed it. It goes against the common sense view of the single background spacetime that QM mistakenly assumes.

Edgar

Edgar L. Owen

unread,
Dec 28, 2013, 1:42:06 PM12/28/13
to everyth...@googlegroups.com
Jason,

Jason,

The categories in your table are largely irrelevant to my theory. You just need to understand the theory.

To answer your first question: Electrons are exact information structures. They don't properly interfere with anything. Their interactions with other things are all exactly computable. It is only as dimensional spacetime emerges from their interactions with whatever, that it seems like they must have interfered with something. But as I noted that is not an observation, it's an inference. Particles acting as waves are NEVER observed directly. Only particles acting as particles are actually observable as they decohere with other things and generate precise dimensional attributes.

Thus the so called 'wave nature' of particles is not real in the strict sense because it's never observable. Particles look like they must have behaved like waves only relative to some dimensional frame they are not native to and hadn't decohered with.

Edgar

meekerdb

unread,
Dec 28, 2013, 4:39:18 PM12/28/13
to everyth...@googlegroups.com
On 12/28/2013 4:54 AM, Edgar L. Owen wrote:
Jason,

Clock time is emergent from comp but comp takes place sequentially in P-time, which is effectively the processor cycles of comp.

No, the computational steps have nothing to do with the computed time.  Just as when I run a Monte Carlo simulation of some events occuring as a Poisson process I can use a random number generator to produce the event times, but they are not produced in the order they occur in simulated time.

Brent

Edgar L. Owen

unread,
Dec 28, 2013, 4:46:30 PM12/28/13
to everyth...@googlegroups.com
Brent,

Maybe in your theory of reality but not in mine...

Edgar

Jason Resch

unread,
Dec 28, 2013, 4:46:08 PM12/28/13
to everyth...@googlegroups.com


On Dec 28, 2013, at 10:11 AM, spudb...@aol.com wrote:

One interpretation by some physicists with Cramer's transactional model, implies that information is coming from the future, and handshaking with the paste to create the present. Price's old book seems to imply this as well.

Cramer's transactional interpretation is non-local.

Jason

meekerdb

unread,
Dec 28, 2013, 6:15:04 PM12/28/13
to everyth...@googlegroups.com
On 12/28/2013 1:46 PM, Jason Resch wrote:


On Dec 28, 2013, at 10:11 AM, spudb...@aol.com wrote:

One interpretation by some physicists with Cramer's transactional model, implies that information is coming from the future, and handshaking with the paste to create the present. Price's old book seems to imply this as well.

Cramer's transactional interpretation is non-local.

Not really.  It's slower-than-light, but retro.

Brent

meekerdb

unread,
Dec 28, 2013, 6:16:23 PM12/28/13
to everyth...@googlegroups.com
It's not my theory, it's Bruno's.  But in my reality I have often run simulations in which the computed time of events was not in the same order as the time of their computation.

Brent
--

meekerdb

unread,
Dec 28, 2013, 7:12:46 PM12/28/13
to everyth...@googlegroups.com
On 12/27/2013 10:31 PM, Jason Resch wrote:
To that I would add the purely epistemic "non-intepretation" of Peres and Fuchs.
 
"No interpretation needed" -- I can interpret this in two ways, one way is to just take the math and equations literally (this leads to Everett), the other is "shut up and calculate", which leads no where really.


 

 
2. Determined by which observer? The cat is always either dead or alive. It's just a matter of someone making a measurement to find out.

So are you saying that before the measurement the cat is neither alive nor dead, both alive and dead, or definitely alive or definitely dead?  If you, (and I think you are), saying that the cat is always definitely alive or definitely dead, then about about the radioactive atom? Is it ever in a state of being decayed and not decayed? If you say no, it sounds like you are denying the reality of the superposition, which some interpretations do, but then this leads to difficulties explaining how quantum computers work (which require the superposition to exist).

Superposition is just a question of basis.  An eigenstate in one basis is a superposition in another.


Can you provide a concrete example where some system can simultaneously be considered to be both in a superposition and not?  Is this like the superposition having collapsed for Wigner's friend while remaining for Wigner before he enters the room?
 

?? Every pure state can be written as a superposition of a complete set of basis states - that's just Hilbert space math. 

The collapse for Wigner's friend can be interpreted either epistemically or by MWI.

Brent
Anny: What happened to that poor cat? It looks half dead.
Erwin: I don't know. Ask Wigner.
Eugene: I just looked in and it collapsed!

meekerdb

unread,
Dec 28, 2013, 7:24:13 PM12/28/13
to everyth...@googlegroups.com
Right.  This is the same as the idea put forward by Vic Stenger in "Timeless Reality" and Elizur and Dolev in the paper I cited.  Information travels both ways along a particle worldline - which is consistent with the time symmetry of the equations.

Brent

Jason Resch

unread,
Dec 28, 2013, 7:45:12 PM12/28/13
to Everything List
On Sat, Dec 28, 2013 at 7:12 PM, meekerdb <meek...@verizon.net> wrote:
On 12/27/2013 10:31 PM, Jason Resch wrote:
To that I would add the purely epistemic "non-intepretation" of Peres and Fuchs.
 
"No interpretation needed" -- I can interpret this in two ways, one way is to just take the math and equations literally (this leads to Everett), the other is "shut up and calculate", which leads no where really.


 

 
2. Determined by which observer? The cat is always either dead or alive. It's just a matter of someone making a measurement to find out.

So are you saying that before the measurement the cat is neither alive nor dead, both alive and dead, or definitely alive or definitely dead?  If you, (and I think you are), saying that the cat is always definitely alive or definitely dead, then about about the radioactive atom? Is it ever in a state of being decayed and not decayed? If you say no, it sounds like you are denying the reality of the superposition, which some interpretations do, but then this leads to difficulties explaining how quantum computers work (which require the superposition to exist).

Superposition is just a question of basis.  An eigenstate in one basis is a superposition in another.


Can you provide a concrete example where some system can simultaneously be considered to be both in a superposition and not?  Is this like the superposition having collapsed for Wigner's friend while remaining for Wigner before he enters the room?
 

?? Every pure state can be written as a superposition of a complete set of basis states - that's just Hilbert space math. 


So then when is the system not in a superposition?

Jason
 
The collapse for Wigner's friend can be interpreted either epistemically or by MWI.

Brent
Anny: What happened to that poor cat? It looks half dead.
Erwin: I don't know. Ask Wigner.
Eugene: I just looked in and it collapsed!

--

meekerdb

unread,
Dec 28, 2013, 8:32:50 PM12/28/13
to everyth...@googlegroups.com
On 12/28/2013 4:45 PM, Jason Resch wrote:



On Sat, Dec 28, 2013 at 7:12 PM, meekerdb <meek...@verizon.net> wrote:
On 12/27/2013 10:31 PM, Jason Resch wrote:
To that I would add the purely epistemic "non-intepretation" of Peres and Fuchs.
 
"No interpretation needed" -- I can interpret this in two ways, one way is to just take the math and equations literally (this leads to Everett), the other is "shut up and calculate", which leads no where really.


 

 
2. Determined by which observer? The cat is always either dead or alive. It's just a matter of someone making a measurement to find out.

So are you saying that before the measurement the cat is neither alive nor dead, both alive and dead, or definitely alive or definitely dead?  If you, (and I think you are), saying that the cat is always definitely alive or definitely dead, then about about the radioactive atom? Is it ever in a state of being decayed and not decayed? If you say no, it sounds like you are denying the reality of the superposition, which some interpretations do, but then this leads to difficulties explaining how quantum computers work (which require the superposition to exist).

Superposition is just a question of basis.  An eigenstate in one basis is a superposition in another.


Can you provide a concrete example where some system can simultaneously be considered to be both in a superposition and not?  Is this like the superposition having collapsed for Wigner's friend while remaining for Wigner before he enters the room?
 

?? Every pure state can be written as a superposition of a complete set of basis states - that's just Hilbert space math. 


So then when is the system not in a superposition?

When it's an incoherent mixture of pure states.

Brent

Jason Resch

unread,
Dec 28, 2013, 9:41:18 PM12/28/13
to Everything List
What makes it incoherent though?  An electron in a superposition, when measured, is still in a superposition according to MWI. It is just that the person doing the measurement is now also caught up in that superposition.

The only thing that can destroy this superposition is to move everything back into the same state it was originally for all the possible diverged states, which should practically never happen for a superposition that has leaked into the environment.

Jason

meekerdb

unread,
Dec 29, 2013, 1:47:40 AM12/29/13
to everyth...@googlegroups.com
On 12/28/2013 6:41 PM, Jason Resch wrote:



On Sat, Dec 28, 2013 at 8:32 PM, meekerdb <meek...@verizon.net> wrote:
On 12/28/2013 4:45 PM, Jason Resch wrote:



On Sat, Dec 28, 2013 at 7:12 PM, meekerdb <meek...@verizon.net> wrote:
On 12/27/2013 10:31 PM, Jason Resch wrote:
To that I would add the purely epistemic "non-intepretation" of Peres and Fuchs.
 
"No interpretation needed" -- I can interpret this in two ways, one way is to just take the math and equations literally (this leads to Everett), the other is "shut up and calculate", which leads no where really.


 

 
2. Determined by which observer? The cat is always either dead or alive. It's just a matter of someone making a measurement to find out.

So are you saying that before the measurement the cat is neither alive nor dead, both alive and dead, or definitely alive or definitely dead?  If you, (and I think you are), saying that the cat is always definitely alive or definitely dead, then about about the radioactive atom? Is it ever in a state of being decayed and not decayed? If you say no, it sounds like you are denying the reality of the superposition, which some interpretations do, but then this leads to difficulties explaining how quantum computers work (which require the superposition to exist).

Superposition is just a question of basis.  An eigenstate in one basis is a superposition in another.


Can you provide a concrete example where some system can simultaneously be considered to be both in a superposition and not?  Is this like the superposition having collapsed for Wigner's friend while remaining for Wigner before he enters the room?
 

?? Every pure state can be written as a superposition of a complete set of basis states - that's just Hilbert space math. 


So then when is the system not in a superposition?

When it's an incoherent mixture of pure states. 

What makes it incoherent though? 

If the density matrix is not a projection operator, i.e. rho^2 =/= rho, it's incoherent.

But really I just meant that in theory there is a basis in which any given pure state is just (1,0,0,...).  In theory there is a 'dead&alive' basis in which Schrodinger's cat can be represented just like a spin-up state is a superposition is a spin-left basis.


An electron in a superposition, when measured, is still in a superposition according to MWI. It is just that the person doing the measurement is now also caught up in that superposition.

The only thing that can destroy this superposition is to move everything back into the same state it was originally for all the possible diverged states, which should practically never happen for a superposition that has leaked into the environment.

In Everett's interpretation a pure state can never evolve into a mixture because the evolution is via a Hermitian operator, the Hamiltonian.  Decoherence makes the submatrix corresponding to the system+instrument to approximate a mixture.  That's why it can be interpreted as giving classical probabilities.

Brent

Jason

John Clark

unread,
Dec 29, 2013, 3:05:24 PM12/29/13
to everyth...@googlegroups.com
On Fri, Dec 27, 2013 at 10:08 PM, Edgar L. Owen <edga...@att.net> wrote

>>  Are faster-than-light influences involved?

>  No.

That means you think things are local.

>> 2. When it is determined whether or not Schrodinger's cat is alive or dead?

>>  The cat is always either dead or alive. It's just a matter of someone making a measurement to find out.

That means you think things are realistic, and that means  I know for a fact your thinking is wrong, not crazy but wrong. We know from experiment that Bell's inequality is violated, and that means that locality or realism or both MUST be wrong. And yes I know that's crazy, but complain to the universe not to me. Your ideas are not crazy, and that is exactly why they're wrong. If I were making a universe I'd make it your way too, but unfortunately Yehowah got the job not me.

  John K Clark



John Clark

unread,
Dec 29, 2013, 3:28:10 PM12/29/13
to everyth...@googlegroups.com
On Sat, Dec 28, 2013 at 6:15 PM, meekerdb <meek...@verizon.net> wrote:

>> Cramer's transactional interpretation is non-local.
> Not really.  It's slower-than-light, but retro.

If you can reach the finish line of a race before you even hear the starting gun I'd say you're pretty damn fast.

From Wikipedia:

"The transactional interpretation of quantum mechanics is explicitly non-local [...]  As such it incorporates the non-locality demonstrated by the Bell test experiments and eliminates the observer dependent reality that plagues the Copenhagen Interpretation"

  John K Clark

 

LizR

unread,
Dec 29, 2013, 3:29:21 PM12/29/13
to everyth...@googlegroups.com
Not quite, violations of Bell's inequality can also be explained by time symmetry (Huw Price and John Bell, private communications).


John Clark

unread,
Dec 29, 2013, 3:42:04 PM12/29/13
to everyth...@googlegroups.com
On Sun, Dec 29, 2013 at 3:29 PM, LizR <liz...@gmail.com> wrote:

> violations of Bell's inequality can also be explained by time symmetry (Huw Price and John Bell, private communications).

I have no idea what that private communication is, but I do know that time is NOT symmetric.

  John K Clark

Edgar L. Owen

unread,
Dec 29, 2013, 4:06:48 PM12/29/13
to everyth...@googlegroups.com
John,

No. See the explanation in my new topic "Another shot at how spacetime emerges from computational reality" and you will (hopefully) see why those problems are avoided...

Edgar

meekerdb

unread,
Dec 29, 2013, 4:09:09 PM12/29/13
to everyth...@googlegroups.com
On 12/29/2013 12:28 PM, John Clark wrote:
On Sat, Dec 28, 2013 at 6:15 PM, meekerdb <meek...@verizon.net> wrote:

>> Cramer's transactional interpretation is non-local.
> Not really.  It's slower-than-light, but retro.

If you can reach the finish line of a race before you even hear the starting gun I'd say you're pretty damn fast.

If an influence can go backward in time as well as forward then it can effectively have FTL influence, as in the EPR diagram in the Elitzur/Dolev paper:



Brent


From Wikipedia:

"The transactional interpretation of quantum mechanics is explicitly non-local [...]  As such it incorporates the non-locality demonstrated by the Bell test experiments and eliminates the observer dependent reality that plagues the Copenhagen Interpretation"

  John K Clark

 

Jason Resch

unread,
Dec 29, 2013, 4:49:18 PM12/29/13
to Everything List
On Sun, Dec 29, 2013 at 4:09 PM, meekerdb <meek...@verizon.net> wrote:
On 12/29/2013 12:28 PM, John Clark wrote:
On Sat, Dec 28, 2013 at 6:15 PM, meekerdb <meek...@verizon.net> wrote:

>> Cramer's transactional interpretation is non-local.
> Not really.  It's slower-than-light, but retro.

If you can reach the finish line of a race before you even hear the starting gun I'd say you're pretty damn fast.

If an influence can go backward in time as well as forward then it can effectively have FTL influence, as in the EPR diagram in the Elitzur/Dolev paper:



Brent



Brent and Liz, and others,

I see how this can work without FTL when the entangled pair is fundamentally the same particle (a la, the Feynman-Stueckelberg interpretation of anti-matter).  We can envision the electron measured at Earth to have traveled forwards in time from the EPR source, which we would normally consider as something like a high-energy photon spontaneously decaying into an electron positron pair.  However with the view of anti-matter as normal matter backwards in time, the picture changes to this:

The positron originates in the future at Proxima centauri, travels backwards in time until it is struck by a photon which is absorbed, and deflects the positron in time so that it travels forwards in time (thus making it seem like an electron) in its path towards Earth.  The fact that it was measured in Proxima Centauri in the future preceeded it being hit by the photon and being measured a second time on Earth, and hence we find the normal quantum correlations.

My question is: how does this work when the entanglement is NOT the result of a particle decay, and so the entanglement cannot be considered the same particle?  Are such entanglements possible or do they all involve cases that under the lens of Feynman-Stueckelberg, may be viewed as the same particle?

Is this the same point raised by: http://en.wikipedia.org/wiki/Transactional_interpretation ?

5. It is not clear how the transactional interpretation handles the quantum mechanics of more than one particle.

    • Daniel F. Styer, Miranda S. Balkin, Kathryn M. Becker, Matthew R. Burns, Christopher E. Dudley, Scott T. Forth, Jeremy S. Gaumer, Mark A. Kramer, David C. Oertel, Leonard H. Park, Marie T. Rinkoski, Clait T. Smith and Timothy D. Wotherspoon (2002) "Nine formulations of quantum mechanics," American Journal of Physics 70, 288-297.
Or is that something else?

Jason
egjgdbec.png

Jason Resch

unread,
Dec 29, 2013, 4:52:22 PM12/29/13
to Everything List
On Sun, Dec 29, 2013 at 3:05 PM, John Clark <johnk...@gmail.com> wrote:
On Fri, Dec 27, 2013 at 10:08 PM, Edgar L. Owen <edga...@att.net> wrote

>>  Are faster-than-light influences involved?

>  No.

That means you think things are local.

>> 2. When it is determined whether or not Schrodinger's cat is alive or dead?

>>  The cat is always either dead or alive. It's just a matter of someone making a measurement to find out.

That means you think things are realistic, and that means  I know for a fact your thinking is wrong, not crazy but wrong. We know from experiment that Bell's inequality is violated, and that means that locality or realism or both MUST be wrong.

Or measurements are multi-valued.  MWI has both locality and realism.

Of course it is still crazy. :-)

Jason
 
And yes I know that's crazy, but complain to the universe not to me. Your ideas are not crazy, and that is exactly why they're wrong. If I were making a universe I'd make it your way too, but unfortunately Yehowah got the job not me.

  John K Clark



Jason Resch

unread,
Dec 29, 2013, 5:01:51 PM12/29/13
to Everything List
On Sun, Dec 29, 2013 at 1:47 AM, meekerdb <meek...@verizon.net> wrote:
On 12/28/2013 6:41 PM, Jason Resch wrote:



On Sat, Dec 28, 2013 at 8:32 PM, meekerdb <meek...@verizon.net> wrote:
On 12/28/2013 4:45 PM, Jason Resch wrote:



On Sat, Dec 28, 2013 at 7:12 PM, meekerdb <meek...@verizon.net> wrote:
On 12/27/2013 10:31 PM, Jason Resch wrote:
To that I would add the purely epistemic "non-intepretation" of Peres and Fuchs.
 
"No interpretation needed" -- I can interpret this in two ways, one way is to just take the math and equations literally (this leads to Everett), the other is "shut up and calculate", which leads no where really.


 

 
2. Determined by which observer? The cat is always either dead or alive. It's just a matter of someone making a measurement to find out.

So are you saying that before the measurement the cat is neither alive nor dead, both alive and dead, or definitely alive or definitely dead?  If you, (and I think you are), saying that the cat is always definitely alive or definitely dead, then about about the radioactive atom? Is it ever in a state of being decayed and not decayed? If you say no, it sounds like you are denying the reality of the superposition, which some interpretations do, but then this leads to difficulties explaining how quantum computers work (which require the superposition to exist).

Superposition is just a question of basis.  An eigenstate in one basis is a superposition in another.


Can you provide a concrete example where some system can simultaneously be considered to be both in a superposition and not?  Is this like the superposition having collapsed for Wigner's friend while remaining for Wigner before he enters the room?
 

?? Every pure state can be written as a superposition of a complete set of basis states - that's just Hilbert space math. 


So then when is the system not in a superposition?

When it's an incoherent mixture of pure states. 

What makes it incoherent though? 

If the density matrix is not a projection operator, i.e. rho^2 =/= rho, it's incoherent.

But really I just meant that in theory there is a basis in which any given pure state is just (1,0,0,...).  In theory there is a 'dead&alive' basis in which Schrodinger's cat can be represented just like a spin-up state is a superposition is a spin-left basis.


So if someone keeps alternating between measuring the spin on the y axis, and then the spin on the x axis, are they not multiplying themselves continuously into diverging states (under MWI)?  Even though these states only weakly interfere, are they not still superposed (that is, the particles involved in a simultaneous combination of possessing many different states for their properties)?
 

An electron in a superposition, when measured, is still in a superposition according to MWI. It is just that the person doing the measurement is now also caught up in that superposition.

The only thing that can destroy this superposition is to move everything back into the same state it was originally for all the possible diverged states, which should practically never happen for a superposition that has leaked into the environment.

In Everett's interpretation a pure state can never evolve into a mixture because the evolution is via a Hermitian operator, the Hamiltonian.  Decoherence makes the submatrix corresponding to the system+instrument to approximate a mixture.  That's why it can be interpreted as giving classical probabilities.

Are there pure states in Everett's interpretation? Doesn't one have to consider the wave function of the universe and consider it all the way into the past?

In any case, returning to the original point that began this tangent, do agree that QM interpretations which are anti-realist (or deny the reality of the superposition) are unable to describe where the intermediate computations that produce the answer to a quantum computation, take place?

What would Fuchs say about quantum computation?

Jason
 

meekerdb

unread,
Dec 29, 2013, 5:29:07 PM12/29/13
to everyth...@googlegroups.com
On 12/29/2013 2:01 PM, Jason Resch wrote:



On Sun, Dec 29, 2013 at 1:47 AM, meekerdb <meek...@verizon.net> wrote:
On 12/28/2013 6:41 PM, Jason Resch wrote:



On Sat, Dec 28, 2013 at 8:32 PM, meekerdb <meek...@verizon.net> wrote:
On 12/28/2013 4:45 PM, Jason Resch wrote:



On Sat, Dec 28, 2013 at 7:12 PM, meekerdb <meek...@verizon.net> wrote:
On 12/27/2013 10:31 PM, Jason Resch wrote:
To that I would add the purely epistemic "non-intepretation" of Peres and Fuchs.
 
"No interpretation needed" -- I can interpret this in two ways, one way is to just take the math and equations literally (this leads to Everett), the other is "shut up and calculate", which leads no where really.


 

 
2. Determined by which observer? The cat is always either dead or alive. It's just a matter of someone making a measurement to find out.

So are you saying that before the measurement the cat is neither alive nor dead, both alive and dead, or definitely alive or definitely dead?  If you, (and I think you are), saying that the cat is always definitely alive or definitely dead, then about about the radioactive atom? Is it ever in a state of being decayed and not decayed? If you say no, it sounds like you are denying the reality of the superposition, which some interpretations do, but then this leads to difficulties explaining how quantum computers work (which require the superposition to exist).

Superposition is just a question of basis.  An eigenstate in one basis is a superposition in another.


Can you provide a concrete example where some system can simultaneously be considered to be both in a superposition and not?  Is this like the superposition having collapsed for Wigner's friend while remaining for Wigner before he enters the room?
 

?? Every pure state can be written as a superposition of a complete set of basis states - that's just Hilbert space math. 


So then when is the system not in a superposition?

When it's an incoherent mixture of pure states. 

What makes it incoherent though? 

If the density matrix is not a projection operator, i.e. rho^2 =/= rho, it's incoherent.

But really I just meant that in theory there is a basis in which any given pure state is just (1,0,0,...).  In theory there is a 'dead&alive' basis in which Schrodinger's cat can be represented just like a spin-up state is a superposition is a spin-left basis.


So if someone keeps alternating between measuring the spin on the y axis, and then the spin on the x axis, are they not multiplying themselves continuously into diverging states (under MWI)?  Even though these states only weakly interfere, are they not still superposed (that is, the particles involved in a simultaneous combination of possessing many different states for their properties)?

Right, according to Everett, the world state becomes a superposition of states of the form |x0,x1,...> where each xi is either +x, -x, +y, or -y.  And per the Bucky Ball, Young's slit experiment, the spins don't have to observed by anyone.  If the silver atom just goes thru the Stern-Gerlach apparatus and hits the laboratory wall, the superposition is still created.  If it just goes out the window and into space...it's not so clear.


 

An electron in a superposition, when measured, is still in a superposition according to MWI. It is just that the person doing the measurement is now also caught up in that superposition.

The only thing that can destroy this superposition is to move everything back into the same state it was originally for all the possible diverged states, which should practically never happen for a superposition that has leaked into the environment.

In Everett's interpretation a pure state can never evolve into a mixture because the evolution is via a Hermitian operator, the Hamiltonian.  Decoherence makes the submatrix corresponding to the system+instrument to approximate a mixture.  That's why it can be interpreted as giving classical probabilities.

Are there pure states in Everett's interpretation? Doesn't one have to consider the wave function of the universe and consider it all the way into the past?

I suppose the universe could have started in a mixed state, but most cosmologists would invoke Ockham and assume it started in a pure state - which, assuming only unitary evolution, means it's still in a pure state.  Of course since inflation there can be entanglements across event horizons, so FAPP that creates mixed states.



In any case, returning to the original point that began this tangent, do agree that QM interpretations which are anti-realist (or deny the reality of the superposition) are unable to describe where the intermediate computations that produce the answer to a quantum computation, take place?

They take place in a quantum computer.



What would Fuchs say about quantum computation?

It's a physical process whose outcome is predicted by QM.

Brent

Jason Resch

unread,
Dec 29, 2013, 6:31:52 PM12/29/13
to Everything List
And the quantum computer is a coherent, long-lived superposition with a number of real states exponential with the number of its qubits.  If superpositions are real and long-lived, and involve an arbitrary number of particles, it seems there is no reason that people could not also be in superpositions.
 


What would Fuchs say about quantum computation?

It's a physical process whose outcome is predicted by QM.




We limit the power and effectiveness our own theories and stifle progress, when we don't put forward theories that make bold statements about reality.  Bohr's (and seemingly Fuch's) positions are so conservative as to never be falsified, but they also inhibit progress and new understandings. For example, general purpose quantum computers may not have been invented had Deutsch not been operating under Everett's paradigm.

"You are the only contemporary physicist, besides Laue, who sees that one cannot get around the assumption of reality, if only one is honest. Most of them simply do not see what sort of risky game they are playing with reality—reality as something independent of what is experimentally established." --- Einstein in a letter to Schrodinger

Jason

meekerdb

unread,
Dec 29, 2013, 6:52:34 PM12/29/13
to everyth...@googlegroups.com
On 12/29/2013 3:31 PM, Jason Resch wrote:
On Sun, Dec 29, 2013 at 5:29 PM, meekerdb <meek...@verizon.net> wrote:
On 12/29/2013 2:01 PM, Jason Resch wrote:
On Sun, Dec 29, 2013 at 1:47 AM, meekerdb <meek...@verizon.net> wrote:
On 12/28/2013 6:41 PM, Jason Resch wrote:
On Sat, Dec 28, 2013 at 8:32 PM, meekerdb <meek...@verizon.net> wrote:
On 12/28/2013 4:45 PM, Jason Resch wrote:
On Sat, Dec 28, 2013 at 7:12 PM, meekerdb <meek...@verizon.net> wrote:
On 12/27/2013 10:31 PM, Jason Resch wrote:
To that I would add the purely epistemic "non-intepretation" of Peres and Fuchs.
�
"No interpretation needed" -- I can interpret this in two ways, one way is to just take the math and equations literally (this leads to Everett), the other is "shut up and calculate", which leads no where really.


�

�
2. Determined by which observer? The cat is always either dead or alive. It's just a matter of someone making a measurement to find out.

So are you saying that before the measurement the cat is neither alive nor dead, both alive and dead, or definitely alive or definitely dead? �If you, (and I think you are), saying that the cat is always definitely alive or definitely dead, then about about the radioactive atom? Is it ever in a state of being decayed and not decayed? If you say no, it sounds like you are denying the reality of the superposition, which some interpretations do, but then this leads to difficulties explaining how quantum computers work (which require the superposition to exist).

Superposition is just a question of basis.� An eigenstate in one basis is a superposition in another.


Can you provide a concrete example where some system can simultaneously be considered to be both in a superposition and not? �Is this like the superposition having collapsed for Wigner's friend while remaining for Wigner before he enters the room?
�

?? Every pure state can be written as a superposition of a complete set of basis states - that's just Hilbert space math.�


So then when is the system not in a superposition?

When it's an incoherent mixture of pure states.�

What makes it incoherent though?�

If the density matrix is not a projection operator, i.e. rho^2 =/= rho, it's incoherent.

But really I just meant that in theory there is a basis in which any given pure state is just (1,0,0,...).� In theory there is a 'dead&alive' basis in which Schrodinger's cat can be represented just like a spin-up state is a superposition is a spin-left basis.


So if someone keeps alternating between measuring the spin on the y axis, and then the spin on the x axis, are they not multiplying themselves continuously into diverging states (under MWI)? �Even though these states only weakly interfere, are they not still superposed (that is, the particles involved in a simultaneous combination of possessing many different states for their properties)?

Right, according to Everett, the world state becomes a superposition of states of the form |x0,x1,...> where each xi is either +x, -x, +y, or -y.� And per the Bucky Ball, Young's slit experiment, the spins don't have to observed by anyone.� If the silver atom just goes thru the Stern-Gerlach apparatus and hits the laboratory wall, the superposition is still created.� If it just goes out the window and into space...it's not so clear.


�

An electron in a superposition, when measured, is still in a superposition according to MWI. It is just that the person doing the measurement is now also caught up in that superposition.

The only thing that can destroy this superposition is to move everything back into the same state it was originally for all the possible diverged states, which should practically never happen for a superposition that has leaked into the environment.

In Everett's interpretation a pure state can never evolve into a mixture because the evolution is via a Hermitian operator, the Hamiltonian.� Decoherence makes the submatrix corresponding to the system+instrument to approximate a mixture.� That's why it can be interpreted as giving classical probabilities.

Are there pure states in Everett's interpretation? Doesn't one have to consider the wave function of the universe and consider it all the way into the past?

I suppose the universe could have started in a mixed state, but most cosmologists would invoke Ockham and assume it started in a pure state - which, assuming only unitary evolution, means it's still in a pure state.� Of course since inflation there can be entanglements across event horizons, so FAPP that creates mixed states.



In any case, returning to the original point that began this tangent, do agree that QM interpretations which are anti-realist (or deny the reality of the superposition) are unable to describe where the intermediate computations that produce the answer to a quantum computation, take place?

They take place in a quantum computer.


And the quantum computer is a coherent, long-lived superposition with a number of real states exponential with the number of its qubits.�

I'm not sure what you mean by "a number of real states"?� It has only one state (which is in a complex Hilbert space), which can be written as a superposition of some set of basis states - but that's true of my refrigerator too.


If superpositions are real and long-lived, and involve an arbitrary number of particles, it seems there is no reason that people could not also be in superpositions.
�


What would Fuchs say about quantum computation?

It's a physical process whose outcome is predicted by QM.




We limit the power and effectiveness our own theories and stifle progress, when we don't put forward theories that make bold statements about reality.�

And we divert progress when we adopt intuitively appealing theories with no operational content and try to reify them.


Bohr's (and seemingly Fuch's) positions are so conservative as to never be falsified,

Nevertheless they both published more papers than Everett (whose interpretation doesn't seem testable either - if it were, it would be a theory instead of an intepretation).


but they also inhibit progress and new understandings. For example, general purpose quantum computers may not have been invented had Deutsch not been operating under Everett's paradigm.

Feynman wrote about quantum computation well before Deutsch.


"You are the only contemporary physicist, besides�Laue, who sees that one cannot get around the assumption of reality, if only one is honest. Most of them simply do not see what sort of risky game they are playing with reality�reality as something independent of what is experimentally established." --- Einstein in a letter to Schrodinger


Everybody believes in reality. Nobody agrees on what it is. :-)

Brent

Jason Resch

unread,
Dec 29, 2013, 9:59:26 PM12/29/13
to Everything List
On Sun, Dec 29, 2013 at 6:52 PM, meekerdb <meek...@verizon.net> wrote:
On 12/29/2013 3:31 PM, Jason Resch wrote:
On Sun, Dec 29, 2013 at 5:29 PM, meekerdb <meek...@verizon.net> wrote:
On 12/29/2013 2:01 PM, Jason Resch wrote:
On Sun, Dec 29, 2013 at 1:47 AM, meekerdb <meek...@verizon.net> wrote:
On 12/28/2013 6:41 PM, Jason Resch wrote:
On Sat, Dec 28, 2013 at 8:32 PM, meekerdb <meek...@verizon.net> wrote:
On 12/28/2013 4:45 PM, Jason Resch wrote:
On Sat, Dec 28, 2013 at 7:12 PM, meekerdb <meek...@verizon.net> wrote:
On 12/27/2013 10:31 PM, Jason Resch wrote:
To that I would add the purely epistemic "non-intepretation" of Peres and Fuchs.
 
"No interpretation needed" -- I can interpret this in two ways, one way is to just take the math and equations literally (this leads to Everett), the other is "shut up and calculate", which leads no where really.


 

 
2. Determined by which observer? The cat is always either dead or alive. It's just a matter of someone making a measurement to find out.

So are you saying that before the measurement the cat is neither alive nor dead, both alive and dead, or definitely alive or definitely dead?  If you, (and I think you are), saying that the cat is always definitely alive or definitely dead, then about about the radioactive atom? Is it ever in a state of being decayed and not decayed? If you say no, it sounds like you are denying the reality of the superposition, which some interpretations do, but then this leads to difficulties explaining how quantum computers work (which require the superposition to exist).

Superposition is just a question of basis.  An eigenstate in one basis is a superposition in another.


Can you provide a concrete example where some system can simultaneously be considered to be both in a superposition and not?  Is this like the superposition having collapsed for Wigner's friend while remaining for Wigner before he enters the room?
 

?? Every pure state can be written as a superposition of a complete set of basis states - that's just Hilbert space math. 

So then when is the system not in a superposition?
When it's an incoherent mixture of pure states. 

What makes it incoherent though? 
If the density matrix is not a projection operator, i.e. rho^2 =/= rho, it's incoherent.

But really I just meant that in theory there is a basis in which any given pure state is just (1,0,0,...).  In theory there is a 'dead&alive' basis in which Schrodinger's cat can be represented just like a spin-up state is a superposition is a spin-left basis.


So if someone keeps alternating between measuring the spin on the y axis, and then the spin on the x axis, are they not multiplying themselves continuously into diverging states (under MWI)?  Even though these states only weakly interfere, are they not still superposed (that is, the particles involved in a simultaneous combination of possessing many different states for their properties)?

Right, according to Everett, the world state becomes a superposition of states of the form |x0,x1,...> where each xi is either +x, -x, +y, or -y.  And per the Bucky Ball, Young's slit experiment, the spins don't have to observed by anyone.  If the silver atom just goes thru the Stern-Gerlach apparatus and hits the laboratory wall, the superposition is still created.  If it just goes out the window and into space...it's not so clear.


 

An electron in a superposition, when measured, is still in a superposition according to MWI. It is just that the person doing the measurement is now also caught up in that superposition.

The only thing that can destroy this superposition is to move everything back into the same state it was originally for all the possible diverged states, which should practically never happen for a superposition that has leaked into the environment.

In Everett's interpretation a pure state can never evolve into a mixture because the evolution is via a Hermitian operator, the Hamiltonian.  Decoherence makes the submatrix corresponding to the system+instrument to approximate a mixture.  That's why it can be interpreted as giving classical probabilities.

Are there pure states in Everett's interpretation? Doesn't one have to consider the wave function of the universe and consider it all the way into the past?

I suppose the universe could have started in a mixed state, but most cosmologists would invoke Ockham and assume it started in a pure state - which, assuming only unitary evolution, means it's still in a pure state.  Of course since inflation there can be entanglements across event horizons, so FAPP that creates mixed states.



In any case, returning to the original point that began this tangent, do agree that QM interpretations which are anti-realist (or deny the reality of the superposition) are unable to describe where the intermediate computations that produce the answer to a quantum computation, take place?

They take place in a quantum computer.


And the quantum computer is a coherent, long-lived superposition with a number of real states exponential with the number of its qubits. 

I'm not sure what you mean by "a number of real states"?  It has only one state (which is in a complex Hilbert space), which can be written as a superposition of some set of basis states - but that's true of my refrigerator too.


I mean each particle could be considered to possess properties with more than one value (for position, spin, momentum, etc.)
 


If superpositions are real and long-lived, and involve an arbitrary number of particles, it seems there is no reason that people could not also be in superpositions.
 
What would Fuchs say about quantum computation?
It's a physical process whose outcome is predicted by QM.




We limit the power and effectiveness our own theories and stifle progress, when we don't put forward theories that make bold statements about reality. 
And we divert progress when we adopt intuitively appealing theories with no operational content and try to reify them.


That is the only way to make progress.  Propose theories, and falsify them.  Ockham says between theories that make equal predictions, simpler ones are better, and it for theories of equal simplicity, ones that can explain more are also better.  Anti-realist interpretations of QM have no adequate explanation for quantum computers.  They say "don't ask" on fundamental questions, which is never a good attitude to have in science.
 

Bohr's (and seemingly Fuch's) positions are so conservative as to never be falsified,

Nevertheless they both published more papers than Everett (whose interpretation doesn't seem testable either - if it were, it would be a theory instead of an intepretation).

Everett's idea is more properly a theory. It explains the phenomenon of collapse without supposing it is the other ideas of QM that try to interpret what is seen (without offering any explanation for them).

As Deutsch says, we wouldn't call dinosaurs an interpretation of fossils when they are the very thing that explains the appearance of the fossils. So it is with Everett and collapse.
 


but they also inhibit progress and new understandings. For example, general purpose quantum computers may not have been invented had Deutsch not been operating under Everett's paradigm.

Feynman wrote about quantum computation well before Deutsch.


Yes, but not general purpose computers.  And Feynman also used a many-worlds interpretation.
 


"You are the only contemporary physicist, besides Laue, who sees that one cannot get around the assumption of reality, if only one is honest. Most of them simply do not see what sort of risky game they are playing with reality—reality as something independent of what is experimentally established." --- Einstein in a letter to Schrodinger


Everybody believes in reality. Nobody agrees on what it is. :-)

True, but that "reality" is quite a different thing if it is only what is captured by our measurements.

Jason

meekerdb

unread,
Dec 29, 2013, 11:43:22 PM12/29/13
to everyth...@googlegroups.com
On 12/29/2013 6:59 PM, Jason Resch wrote:
That is the only way to make progress. �Propose theories, and falsify them. �Ockham says between theories that make equal predictions, simpler ones are better, and it for theories of equal simplicity, ones that can explain more are also better. �Anti-realist interpretations of QM have no adequate explanation for quantum computers.�

There's nothing "anti-realist" about relational or Bayesian subjective interpretations, they just don't reify the wave function as you would like them to.� Bohm used to make the same complaint that other theories weren't "realistic".� Fuchs et al have as good an explanation of quantum computers as any dynamic quantum system, there's nothing special about computers - it's just not one that appeals to you.


They say "don't ask" on fundamental questions, which is never a good attitude to have in science.

That's your straw man attribution.� You've apparently stopped asking and decided you have the answer.

Brent
The sciences do not try to explain, they hardly even try to� interpret, they mainly make models. By a model is meant a� mathematical construct which, with the addition of certain verbal� interpretations, describes observed phenomena. The justification of� such a mathematical construct is solely and precisely that it is� expected to work.
��� --�John von Neumann

Jason Resch

unread,
Dec 30, 2013, 12:05:17 AM12/30/13
to Everything List
On Sun, Dec 29, 2013 at 11:43 PM, meekerdb <meek...@verizon.net> wrote:
On 12/29/2013 6:59 PM, Jason Resch wrote:
That is the only way to make progress.  Propose theories, and falsify them.  Ockham says between theories that make equal predictions, simpler ones are better, and it for theories of equal simplicity, ones that can explain more are also better.  Anti-realist interpretations of QM have no adequate explanation for quantum computers. 

There's nothing "anti-realist" about relational or Bayesian subjective interpretations, they just don't reify the wave function as you would like them to.  Bohm used to make the same complaint that other theories weren't "realistic".  Fuchs et al have as good an explanation of quantum computers as any dynamic quantum system, there's nothing special about computers - it's just not one that appeals to you.

Computers in particular, while not special, are good examples because they illustrate that nothing known in our universe (aside from the superposition) has the necessarily complexity to produce answers to certain complex problems.
 


They say "don't ask" on fundamental questions, which is never a good attitude to have in science.

That's your straw man attribution.  You've apparently stopped asking and decided you have the answer.

I would rather choose a speculative interpretation that turns out to be wrong then say QM needs no interpretation, nor should we look for one, as the paper you recently cited suggested.
 

Brent
The sciences do not try to explain, they hardly even try to  interpret, they mainly make models. By a model is meant a  mathematical construct which, with the addition of certain verbal  interpretations, describes observed phenomena. The justification of  such a mathematical construct is solely and precisely that it is  expected to work.
    --—John von Neumann

If Fuchs et al operated according to this quote, they would see that a model is not the same thing as the description/predictions of observed phenomena that it makes.

If we identify reality only with observed phenomena, what is to prevent us from falling into solipsism or idealism?
 
Jason

meekerdb

unread,
Dec 30, 2013, 2:49:19 AM12/30/13
to everyth...@googlegroups.com
On 12/29/2013 9:05 PM, Jason Resch wrote:
On Sun, Dec 29, 2013 at 11:43 PM, meekerdb <meek...@verizon.net> wrote:
On 12/29/2013 6:59 PM, Jason Resch wrote:
That is the only way to make progress. �Propose theories, and falsify them. �Ockham says between theories that make equal predictions, simpler ones are better, and it for theories of equal simplicity, ones that can explain more are also better. �Anti-realist interpretations of QM have no adequate explanation for quantum computers.�

There's nothing "anti-realist" about relational or Bayesian subjective interpretations, they just don't reify the wave function as you would like them to.� Bohm used to make the same complaint that other theories weren't "realistic".� Fuchs et al have as good an explanation of quantum computers as any dynamic quantum system, there's nothing special about computers - it's just not one that appeals to you.

Computers in particular, while not special, are good examples because they illustrate that nothing known in our universe (aside from the superposition) has the necessarily complexity to produce answers to certain complex problems.

But that's essentially everything, since everything is (presumably) quantum.� But notice the limitation of quantum computers, if it has N qubits it takes 2^N complex numbers to specify its state, BUT you can only retrieve N bits of information from it (c.f. Holevo's theorem).� So it doesn't really act like 2^N parallel computers.

�


They say "don't ask" on fundamental questions, which is never a good attitude to have in science.

That's your straw man attribution.� You've apparently stopped asking and decided you have the answer.

I would rather choose a speculative interpretation that turns out to be wrong then say QM needs no interpretation, nor should we look for one, as the paper you recently cited suggested.
�

Brent

The sciences do not try to explain, they hardly even try to� interpret, they mainly make models. By a model is meant a� mathematical construct which, with the addition of certain verbal� interpretations, describes observed phenomena. The justification of� such a mathematical construct is solely and precisely that it is� expected to work.
��� --�John von Neumann

If Fuchs et al operated according to this quote, they would see that a model is not the same thing as the description/predictions of observed phenomena that it makes.

But it could be.� You only know the observations - you don't know the reality in itself.



If we identify reality only with observed phenomena, what is to prevent us from falling into solipsism or idealism?

Solipism doesn't seem to work well.� When I kick people they kick back.� :-)

Brent
"I'm a Solipist, and I must say I'm surprised there aren't more of us."
����� -- letter to Bertrand Russell

Bruno Marchal

unread,
Dec 30, 2013, 3:12:44 AM12/30/13
to everyth...@googlegroups.com
On 29 Dec 2013, at 21:05, John Clark wrote:

On Fri, Dec 27, 2013 at 10:08 PM, Edgar L. Owen <edga...@att.net> wrote

>>  Are faster-than-light influences involved?

>  No.

That means you think things are local.

>> 2. When it is determined whether or not Schrodinger's cat is alive or dead?

>>  The cat is always either dead or alive. It's just a matter of someone making a measurement to find out.

That means you think things are realistic, and that means  I know for a fact your thinking is wrong, not crazy but wrong. We know from experiment that Bell's inequality is violated, and that means that locality or realism or both MUST be wrong.

Locally. In one branch. But the wave evolves deterministically, and the entire multiverse is local. There are no spooky action at a distance in the many-world picture, imo.



And yes I know that's crazy, but complain to the universe not to me.

Ah Ah .... You talk on the universe like it was your God ....  :)
God is the one you can complain too, indeed, when humans can't listen to your complain ... It can help for the moral.

Bruno


Your ideas are not crazy, and that is exactly why they're wrong. If I were making a universe I'd make it your way too, but unfortunately Yehowah got the job not me.

  John K Clark




--
You received this message because you are subscribed to the Google Groups "Everything List" group.
To unsubscribe from this group and stop receiving emails from it, send an email to everything-li...@googlegroups.com.
To post to this group, send email to everyth...@googlegroups.com.
Visit this group at http://groups.google.com/group/everything-list.
For more options, visit https://groups.google.com/groups/opt_out.

Bruno Marchal

unread,
Dec 30, 2013, 3:24:36 AM12/30/13
to everyth...@googlegroups.com
On 29 Dec 2013, at 21:29, LizR wrote:

Not quite, violations of Bell's inequality can also be explained by time symmetry (Huw Price and John Bell, private communications).

+ very special initial boundary conditions, which leads to a selection principle in the MW. It is a bit like in Bohm, except that the potential is replaced by boundary conditions. Bell was opposed to many-world, and even, before Aspect, he predicted that nature would not violate its inequality. Also, those boundary condition leads to a simulation of an Heinsenberg cut, as all branches must continue to exist at some level, to get the interference effect, and not above that level to get a physical reality unique.  This is logically consistent, I can agree, but seems more ad hoc compared to postulate that QM is "universal". 

We do have something similar with comp. We cannot really prove that the numbers cannot conspire for making a physical reality unique. They might even conspire to make *you* unique.  That would be depressing, no? I would be a zombie!

Bruno

Bruno Marchal

unread,
Dec 30, 2013, 3:27:10 AM12/30/13
to everyth...@googlegroups.com
You cannot know that. You can believe that, because you believe in some facts and theories.

Bruno



  John K Clark


--
You received this message because you are subscribed to the Google Groups "Everything List" group.
To unsubscribe from this group and stop receiving emails from it, send an email to everything-li...@googlegroups.com.
To post to this group, send email to everyth...@googlegroups.com.
Visit this group at http://groups.google.com/group/everything-list.
For more options, visit https://groups.google.com/groups/opt_out.

Bruno Marchal

unread,
Dec 30, 2013, 3:43:24 AM12/30/13
to everyth...@googlegroups.com
On 29 Dec 2013, at 23:29, meekerdb wrote:

On 12/29/2013 2:01 PM, Jason Resch wrote:



On Sun, Dec 29, 2013 at 1:47 AM, meekerdb <meek...@verizon.net> wrote:
On 12/28/2013 6:41 PM, Jason Resch wrote:



On Sat, Dec 28, 2013 at 8:32 PM, meekerdb <meek...@verizon.net> wrote:
On 12/28/2013 4:45 PM, Jason Resch wrote:



On Sat, Dec 28, 2013 at 7:12 PM, meekerdb <meek...@verizon.net> wrote:
On 12/27/2013 10:31 PM, Jason Resch wrote:
To that I would add the purely epistemic "non-intepretation" of Peres and Fuchs.
 
"No interpretation needed" -- I can interpret this in two ways, one way is to just take the math and equations literally (this leads to Everett), the other is "shut up and calculate", which leads no where really.


 

 
2. Determined by which observer? The cat is always either dead or alive. It's just a matter of someone making a measurement to find out.

So are you saying that before the measurement the cat is neither alive nor dead, both alive and dead, or definitely alive or definitely dead?  If you, (and I think you are), saying that the cat is always definitely alive or definitely dead, then about about the radioactive atom? Is it ever in a state of being decayed and not decayed? If you say no, it sounds like you are denying the reality of the superposition, which some interpretations do, but then this leads to difficulties explaining how quantum computers work (which require the superposition to exist).

Superposition is just a question of basis.  An eigenstate in one basis is a superposition in another.


Can you provide a concrete example where some system can simultaneously be considered to be both in a superposition and not?  Is this like the superposition having collapsed for Wigner's friend while remaining for Wigner before he enters the room?
 

?? Every pure state can be written as a superposition of a complete set of basis states - that's just Hilbert space math. 


So then when is the system not in a superposition?

When it's an incoherent mixture of pure states. 

What makes it incoherent though? 

If the density matrix is not a projection operator, i.e. rho^2 =/= rho, it's incoherent.

But really I just meant that in theory there is a basis in which any given pure state is just (1,0,0,...).  In theory there is a 'dead&alive' basis in which Schrodinger's cat can be represented just like a spin-up state is a superposition is a spin-left basis.


So if someone keeps alternating between measuring the spin on the y axis, and then the spin on the x axis, are they not multiplying themselves continuously into diverging states (under MWI)?  Even though these states only weakly interfere, are they not still superposed (that is, the particles involved in a simultaneous combination of possessing many different states for their properties)?

Right, according to Everett, the world state becomes a superposition of states of the form |x0,x1,...> where each xi is either +x, -x, +y, or -y.  And per the Bucky Ball, Young's slit experiment, the spins don't have to observed by anyone.  If the silver atom just goes thru the Stern-Gerlach apparatus and hits the laboratory wall, the superposition is still created.  If it just goes out the window and into space...it's not so clear.

It is very clear. IF QM is exact, the superposition does not disappear, but get contagious to the environment (at roughly the speed of light). 




 

An electron in a superposition, when measured, is still in a superposition according to MWI. It is just that the person doing the measurement is now also caught up in that superposition.

The only thing that can destroy this superposition is to move everything back into the same state it was originally for all the possible diverged states, which should practically never happen for a superposition that has leaked into the environment.

In Everett's interpretation a pure state can never evolve into a mixture because the evolution is via a Hermitian operator, the Hamiltonian.  Decoherence makes the submatrix corresponding to the system+instrument to approximate a mixture.  That's why it can be interpreted as giving classical probabilities.

Are there pure states in Everett's interpretation? Doesn't one have to consider the wave function of the universe and consider it all the way into the past?

I suppose the universe could have started in a mixed state, but most cosmologists would invoke Ockham and assume it started in a pure state - which, assuming only unitary evolution, means it's still in a pure state.  Of course since inflation there can be entanglements across event horizons, so FAPP that creates mixed states.

FAPP. yes.




In any case, returning to the original point that began this tangent, do agree that QM interpretations which are anti-realist (or deny the reality of the superposition) are unable to describe where the intermediate computations that produce the answer to a quantum computation, take place?

They take place in a quantum computer.

That beg the question ...





What would Fuchs say about quantum computation?

It's a physical process whose outcome is predicted by QM.

That looks like instrumentalism and "don't ask for understanding".

Bruno



Bruno Marchal

unread,
Dec 30, 2013, 4:08:42 AM12/30/13
to everyth...@googlegroups.com
On 30 Dec 2013, at 00:52, meekerdb wrote:

On 12/29/2013 3:31 PM, Jason Resch wrote:
On Sun, Dec 29, 2013 at 5:29 PM, meekerdb <meek...@verizon.net> wrote:
On 12/29/2013 2:01 PM, Jason Resch wrote:
On Sun, Dec 29, 2013 at 1:47 AM, meekerdb <meek...@verizon.net> wrote:
On 12/28/2013 6:41 PM, Jason Resch wrote:
On Sat, Dec 28, 2013 at 8:32 PM, meekerdb <meek...@verizon.net> wrote:
On 12/28/2013 4:45 PM, Jason Resch wrote:
On Sat, Dec 28, 2013 at 7:12 PM, meekerdb <meek...@verizon.net> wrote:
On 12/27/2013 10:31 PM, Jason Resch wrote:
To that I would add the purely epistemic "non-intepretation" of Peres and Fuchs.
 
"No interpretation needed" -- I can interpret this in two ways, one way is to just take the math and equations literally (this leads to Everett), the other is "shut up and calculate", which leads no where really.


 

 
2. Determined by which observer? The cat is always either dead or alive. It's just a matter of someone making a measurement to find out.

So are you saying that before the measurement the cat is neither alive nor dead, both alive and dead, or definitely alive or definitely dead?  If you, (and I think you are), saying that the cat is always definitely alive or definitely dead, then about about the radioactive atom? Is it ever in a state of being decayed and not decayed? If you say no, it sounds like you are denying the reality of the superposition, which some interpretations do, but then this leads to difficulties explaining how quantum computers work (which require the superposition to exist).

Superposition is just a question of basis.  An eigenstate in one basis is a superposition in another.


Can you provide a concrete example where some system can simultaneously be considered to be both in a superposition and not?  Is this like the superposition having collapsed for Wigner's friend while remaining for Wigner before he enters the room?
 

?? Every pure state can be written as a superposition of a complete set of basis states - that's just Hilbert space math. 

So then when is the system not in a superposition?
When it's an incoherent mixture of pure states. 

What makes it incoherent though? 
If the density matrix is not a projection operator, i.e. rho^2 =/= rho, it's incoherent.

But really I just meant that in theory there is a basis in which any given pure state is just (1,0,0,...).  In theory there is a 'dead&alive' basis in which Schrodinger's cat can be represented just like a spin-up state is a superposition is a spin-left basis.


So if someone keeps alternating between measuring the spin on the y axis, and then the spin on the x axis, are they not multiplying themselves continuously into diverging states (under MWI)?  Even though these states only weakly interfere, are they not still superposed (that is, the particles involved in a simultaneous combination of possessing many different states for their properties)?

Right, according to Everett, the world state becomes a superposition of states of the form |x0,x1,...> where each xi is either +x, -x, +y, or -y.  And per the Bucky Ball, Young's slit experiment, the spins don't have to observed by anyone.  If the silver atom just goes thru the Stern-Gerlach apparatus and hits the laboratory wall, the superposition is still created.  If it just goes out the window and into space...it's not so clear.


 

An electron in a superposition, when measured, is still in a superposition according to MWI. It is just that the person doing the measurement is now also caught up in that superposition.

The only thing that can destroy this superposition is to move everything back into the same state it was originally for all the possible diverged states, which should practically never happen for a superposition that has leaked into the environment.

In Everett's interpretation a pure state can never evolve into a mixture because the evolution is via a Hermitian operator, the Hamiltonian.  Decoherence makes the submatrix corresponding to the system+instrument to approximate a mixture.  That's why it can be interpreted as giving classical probabilities.

Are there pure states in Everett's interpretation? Doesn't one have to consider the wave function of the universe and consider it all the way into the past?

I suppose the universe could have started in a mixed state, but most cosmologists would invoke Ockham and assume it started in a pure state - which, assuming only unitary evolution, means it's still in a pure state.  Of course since inflation there can be entanglements across event horizons, so FAPP that creates mixed states.



In any case, returning to the original point that began this tangent, do agree that QM interpretations which are anti-realist (or deny the reality of the superposition) are unable to describe where the intermediate computations that produce the answer to a quantum computation, take place?

They take place in a quantum computer.


And the quantum computer is a coherent, long-lived superposition with a number of real states exponential with the number of its qubits. 

I'm not sure what you mean by "a number of real states"?  It has only one state (which is in a complex Hilbert space), which can be written as a superposition of some set of basis states - but that's true of my refrigerator too.


But your fridge cannot exploit the superposition like a quantum computer do. I agree with Deutsch and Jason that a quantum computation is quasi-not explainable without being realist on quantum superposition. In fact I agree with Deutsch that the two slits experiment, with the particles sent one at a time,  provides already a strong evidence for "real physical" (but not necessarily primitive) superposition.





If superpositions are real and long-lived, and involve an arbitrary number of particles, it seems there is no reason that people could not also be in superpositions.
 
What would Fuchs say about quantum computation?
It's a physical process whose outcome is predicted by QM.




We limit the power and effectiveness our own theories and stifle progress, when we don't put forward theories that make bold statements about reality. 
And we divert progress when we adopt intuitively appealing theories with no operational content and try to reify them.

Bohr's (and seemingly Fuch's) positions are so conservative as to never be falsified,

Nevertheless they both published more papers than Everett (whose interpretation doesn't seem testable either - if it were, it would be a theory instead of an intepretation).


Everett abandonned physics.
"the number of publication" cannot be invoked to judge the validity of a logical point or a theory. It measure your willingness to publish or perish, only.





but they also inhibit progress and new understandings. For example, general purpose quantum computers may not have been invented had Deutsch not been operating under Everett's paradigm.

Feynman wrote about quantum computation well before Deutsch.

He was a many-worlder, even if, like François Englert, he was not so much "proud" of that aspect of nature, and never insisted on that aspect of the quantum reality.





"You are the only contemporary physicist, besides Laue, who sees that one cannot get around the assumption of reality, if only one is honest. Most of them simply do not see what sort of risky game they are playing with reality—reality as something independent of what is experimentally established." --- Einstein in a letter to Schrodinger


Everybody believes in reality. Nobody agrees on what it is. :-)

Which makes it an interesting subject for long conversation and  fun among good willing people ... 

Bruno



Bruno Marchal

unread,
Dec 30, 2013, 5:05:57 AM12/30/13
to everyth...@googlegroups.com
On 30 Dec 2013, at 03:59, Jason Resch wrote:




On Sun, Dec 29, 2013 at 6:52 PM, meekerdb <meek...@verizon.net> wrote:
On 12/29/2013 3:31 PM, Jason Resch wrote:


Everett's idea is more properly a theory. It explains the phenomenon of collapse without supposing it is the other ideas of QM that try to interpret what is seen (without offering any explanation for them).

As Deutsch says, we wouldn't call dinosaurs an interpretation of fossils when they are the very thing that explains the appearance of the fossils. So it is with Everett and collapse.


I agree. if people read Everett they will see that he does not talk about a new interpretation of QM, but on a new formulation of QM, which is simply the SWE. 

I consider that "MW interpretation" is already a misleading statement. Everett proposed a new theory, which is exactly Copenhagen minus the collapse.

Then Everett explains in that theory where the beliefs in collapse comes from.

Comp does the same, and explains, constructively, where the belief in the universal wave (or matrix or path) come from.

The conceptual progresses are the following

1)
    - SWE
    - Collapse
    - Dualist theory of mind (unintelligible as such)

2)
    - SWE
    - Comp

3)
    - Comp.




Bruno Marchal

unread,
Dec 30, 2013, 6:09:09 AM12/30/13
to everyth...@googlegroups.com
On 30 Dec 2013, at 08:49, meekerdb wrote:

On 12/29/2013 9:05 PM, Jason Resch wrote:
On Sun, Dec 29, 2013 at 11:43 PM, meekerdb <meek...@verizon.net> wrote:
On 12/29/2013 6:59 PM, Jason Resch wrote:
That is the only way to make progress.  Propose theories, and falsify them.  Ockham says between theories that make equal predictions, simpler ones are better, and it for theories of equal simplicity, ones that can explain more are also better.  Anti-realist interpretations of QM have no adequate explanation for quantum computers. 

There's nothing "anti-realist" about relational or Bayesian subjective interpretations, they just don't reify the wave function as you would like them to.  Bohm used to make the same complaint that other theories weren't "realistic".  Fuchs et al have as good an explanation of quantum computers as any dynamic quantum system, there's nothing special about computers - it's just not one that appeals to you.

Computers in particular, while not special, are good examples because they illustrate that nothing known in our universe (aside from the superposition) has the necessarily complexity to produce answers to certain complex problems.

But that's essentially everything, since everything is (presumably) quantum.  But notice the limitation of quantum computers, if it has N qubits it takes 2^N complex numbers to specify its state, BUT you can only retrieve N bits of information from it (c.f. Holevo's theorem).  So it doesn't really act like 2^N parallel computers.


OK, but nobody pretended the contrary.  You can still extract N bits depending on the 2^N results, by doing some Fourier transfrom on all results obtained in "parallel universes". This means that the 2^N computations have to occur in *some* sense.




 


They say "don't ask" on fundamental questions, which is never a good attitude to have in science.

That's your straw man attribution.  You've apparently stopped asking and decided you have the answer.

I would rather choose a speculative interpretation that turns out to be wrong then say QM needs no interpretation, nor should we look for one, as the paper you recently cited suggested.
 

Brent

The sciences do not try to explain, they hardly even try to  interpret, they mainly make models. By a model is meant a  mathematical construct which, with the addition of certain verbal  interpretations, describes observed phenomena. The justification of  such a mathematical construct is solely and precisely that it is  expected to work.
    --—John von Neumann

If Fuchs et al operated according to this quote, they would see that a model is not the same thing as the description/predictions of observed phenomena that it makes.

But it could be.  You only know the observations - you don't know the reality in itself.



If we identify reality only with observed phenomena, what is to prevent us from falling into solipsism or idealism?

Solipism doesn't seem to work well.  When I kick people they kick back.  :-)

Hmm... I already told you about the (lucid) dream of an Indian master, who enjoyed feeling superior to his dreamed disciples.
But one day (well one night), one of the dreamed people in the audience stood up, and told him "well, if you believe that we are not existing because you dream us, explain me who is waking you up right now, and he stroke him with some wood until ... he woke up!" Kicking back is not an absolute criteria for reality. 

But I agree with you, solipsism, perhaps even in dream, does not work too well. Note, though, that the first person is factually solipsist, even if not doctrinally so, as she can *bet* on others and 3p things, fortunately.

Bruno





Brent
"I'm a Solipist, and I must say I'm surprised there aren't more of us."
      -- letter to Bertrand Russell
--
You received this message because you are subscribed to the Google Groups "Everything List" group.
To unsubscribe from this group and stop receiving emails from it, send an email to everything-li...@googlegroups.com.
To post to this group, send email to everyth...@googlegroups.com.
Visit this group at http://groups.google.com/group/everything-list.
For more options, visit https://groups.google.com/groups/opt_out.

meekerdb

unread,
Dec 30, 2013, 2:00:45 PM12/30/13
to everyth...@googlegroups.com
On 12/30/2013 3:09 AM, Bruno Marchal wrote:
But that's essentially everything, since everything is (presumably) quantum.� But notice the limitation of quantum computers, if it has N qubits it takes 2^N complex numbers to specify its state, BUT you can only retrieve N bits of information from it (c.f. Holevo's theorem).� So it doesn't really act like 2^N parallel computers.


OK, but nobody pretended the contrary. �You can still extract N bits depending on the 2^N results, by doing some Fourier transfrom on all results obtained in "parallel universes". This means that the 2^N computations have to occur in *some* sense.

But they pretend that the number 2^N is so large that it cannot exist in whole universe, much less in that little quantum computer and therefore there must be other worlds which contain these enormous number of bits.� What Holevo's theorem shows is the one can regard all those interference terms as mere calculation fictions in going from N bit inputs to N bit outputs.� It is conceptually no different than doing a calculation in ordinary probability theory: I start with some initial conditions and I introduce a probability distribution and compute a probability for some event.� In that intermediate step I introduced a continuous probability distribution which implies an *infinite* number of bits.� Nobody thinks this requires an infinite number of worlds.

Brent

Jason Resch

unread,
Dec 30, 2013, 2:17:36 PM12/30/13
to Everything List
On Mon, Dec 30, 2013 at 2:00 PM, meekerdb <meek...@verizon.net> wrote:
On 12/30/2013 3:09 AM, Bruno Marchal wrote:
But that's essentially everything, since everything is (presumably) quantum.  But notice the limitation of quantum computers, if it has N qubits it takes 2^N complex numbers to specify its state, BUT you can only retrieve N bits of information from it (c.f. Holevo's theorem).  So it doesn't really act like 2^N parallel computers.


OK, but nobody pretended the contrary.  You can still extract N bits depending on the 2^N results, by doing some Fourier transfrom on all results obtained in "parallel universes". This means that the 2^N computations have to occur in *some* sense.

But they pretend that the number 2^N is so large that it cannot exist in whole universe, much less in that little quantum computer and therefore there must be other worlds which contain these enormous number of bits.  What Holevo's theorem shows is the one can regard all those interference terms as mere calculation fictions in going from N bit inputs to N bit outputs. 

Can such "calculation fictions" support conciousness?  That's the real question.  If they can, then you can't avoid many-worlds (or at least many minds).

Jason

 
It is conceptually no different than doing a calculation in ordinary probability theory: I start with some initial conditions and I introduce a probability distribution and compute a probability for some event.  In that intermediate step I introduced a continuous probability distribution which implies an *infinite* number of bits.  Nobody thinks this requires an infinite number of worlds.

Brent

meekerdb

unread,
Dec 30, 2013, 2:41:12 PM12/30/13
to everyth...@googlegroups.com
On 12/30/2013 11:17 AM, Jason Resch wrote:



On Mon, Dec 30, 2013 at 2:00 PM, meekerdb <meek...@verizon.net> wrote:
On 12/30/2013 3:09 AM, Bruno Marchal wrote:
But that's essentially everything, since everything is (presumably) quantum.  But notice the limitation of quantum computers, if it has N qubits it takes 2^N complex numbers to specify its state, BUT you can only retrieve N bits of information from it (c.f. Holevo's theorem).  So it doesn't really act like 2^N parallel computers.


OK, but nobody pretended the contrary.  You can still extract N bits depending on the 2^N results, by doing some Fourier transfrom on all results obtained in "parallel universes". This means that the 2^N computations have to occur in *some* sense.

But they pretend that the number 2^N is so large that it cannot exist in whole universe, much less in that little quantum computer and therefore there must be other worlds which contain these enormous number of bits.  What Holevo's theorem shows is the one can regard all those interference terms as mere calculation fictions in going from N bit inputs to N bit outputs. 

Can such "calculation fictions" support conciousness?  That's the real question.  If they can, then you can't avoid many-worlds (or at least many minds).

Why is that "the real question"?  Saying yes to the doctor implies that a classical computer can support consciousness.

Brent

Jason Resch

unread,
Dec 30, 2013, 3:04:11 PM12/30/13
to Everything List
Because with computationalism, if a quantum computer runs the computations that support a mind, there would be many resulting conscious states, and first person views.  That we can only access N-bits of a mind from any one world is irrelevant, as all the conscious states exist in the intermediate states, which you call "calculation fictions".

Jason 

John Clark

unread,
Dec 30, 2013, 3:49:14 PM12/30/13
to everyth...@googlegroups.com
On Sun, Dec 29, 2013 at 4:09 PM, meekerdb <meek...@verizon.net> wrote:

> If an influence can go backward in time as well as forward then it can effectively have FTL influence,

We already know for a fact that faster than light influences exist, and this has nothing to do with any theory, it was found experimentally. Of course this does not mean you can sent a message faster than light, matter and energy and information are still limited by the speed of light, but influences can certainly go astronomically faster, probably infinitely faster.

  John K Clark   




meekerdb

unread,
Dec 30, 2013, 3:57:46 PM12/30/13
to everyth...@googlegroups.com
Of course that is assuming the very proposition you're arguing.


That we can only access N-bits of a mind from any one world is irrelevant, as all the conscious states exist in the intermediate states,

That's your story and you're sticking to it.

Brent

John Clark

unread,
Dec 30, 2013, 4:00:52 PM12/30/13
to everyth...@googlegroups.com
On Sun, Dec 29, 2013 at 4:52 PM, Jason Resch <jason...@gmail.com> wrote:

>> That means you think things are realistic, and that means  I know for a fact your thinking is wrong, not crazy but wrong. We know from experiment that Bell's inequality is violated, and that means that locality or realism or both MUST be wrong.

> Or measurements are multi-valued.  MWI has both locality and realism.

If the many World's Theory was local AND realistic we'd know with certainty that it's wrong because any theory that is consistent with experimental results can NOT be both.  But MWI could be true because although it is realistic it is not local. A entire parallel universe as big as our own that you can never go to or even see is about as far from being local as you can get.

  John K Clark




 

Jason Resch

unread,
Dec 30, 2013, 4:29:44 PM12/30/13
to Everything List
No, I am trying to show that given computationalism, there is nothing "fictional" about these computations. They would have very bit the same power to yield consciousness as the computations of a classical computer.  Do you disagree with this?
 

That we can only access N-bits of a mind from any one world is irrelevant, as all the conscious states exist in the intermediate states,

That's your story and you're sticking to it.


Do you disagree?

Jason

Jason Resch

unread,
Dec 30, 2013, 4:34:44 PM12/30/13
to Everything List
On Mon, Dec 30, 2013 at 4:00 PM, John Clark <johnk...@gmail.com> wrote:
On Sun, Dec 29, 2013 at 4:52 PM, Jason Resch <jason...@gmail.com> wrote:

>> That means you think things are realistic, and that means  I know for a fact your thinking is wrong, not crazy but wrong. We know from experiment that Bell's inequality is violated, and that means that locality or realism or both MUST be wrong.

> Or measurements are multi-valued.  MWI has both locality and realism.

If the many World's Theory was local AND realistic we'd know with certainty that it's wrong because any theory that is consistent with experimental results can NOT be both. 

There are at least two possible answers to the bell inequalities:
1. Nonlocal influences
2. Mutliple outcomes for each measurement

If you choose 2, then you don't need 1. You can have multiple outcomes for a measurement and realism. You just can't have only one definite real value, without FTL influences.
 
But MWI could be true because although it is realistic it is not local.

It is local, all QM effects under MW propagate at c or slower.  See the "many worlds FAQ" or the wikipedia table comparing various interpretations.
 
A entire parallel universe as big as our own that you can never go to or even see is about as far from being local as you can get.

Locality has a specific definition in physics, that things are only affected by other things (fields or particles) in direct proximity to each other. It says nothing about the existence of places we can or can't go to.

Jason

meekerdb

unread,
Dec 30, 2013, 4:45:51 PM12/30/13
to everyth...@googlegroups.com
On 12/30/2013 1:29 PM, Jason Resch wrote:



On Mon, Dec 30, 2013 at 3:57 PM, meekerdb <meek...@verizon.net> wrote:
On 12/30/2013 12:04 PM, Jason Resch wrote:



On Mon, Dec 30, 2013 at 2:41 PM, meekerdb <meek...@verizon.net> wrote:
On 12/30/2013 11:17 AM, Jason Resch wrote:



On Mon, Dec 30, 2013 at 2:00 PM, meekerdb <meek...@verizon.net> wrote:
On 12/30/2013 3:09 AM, Bruno Marchal wrote:
But that's essentially everything, since everything is (presumably) quantum.  But notice the limitation of quantum computers, if it has N qubits it takes 2^N complex numbers to specify its state, BUT you can only retrieve N bits of information from it (c.f. Holevo's theorem).  So it doesn't really act like 2^N parallel computers.


OK, but nobody pretended the contrary.  You can still extract N bits depending on the 2^N results, by doing some Fourier transfrom on all results obtained in "parallel universes". This means that the 2^N computations have to occur in *some* sense.

But they pretend that the number 2^N is so large that it cannot exist in whole universe, much less in that little quantum computer and therefore there must be other worlds which contain these enormous number of bits.  What Holevo's theorem shows is the one can regard all those interference terms as mere calculation fictions in going from N bit inputs to N bit outputs. 

Can such "calculation fictions" support conciousness?  That's the real question.  If they can, then you can't avoid many-worlds (or at least many minds).

Why is that "the real question"?  Saying yes to the doctor implies that a classical computer can support consciousness.

Because with computationalism, if a quantum computer runs the computations that support a mind, there would be many resulting conscious states, and first person views. 

Of course that is assuming the very proposition you're arguing.


No, I am trying to show that given computationalism, there is nothing "fictional" about these computations. They would have very bit the same power to yield consciousness as the computations of a classical computer.  Do you disagree with this?

I'm not sure what you mean by "power"; whether it means effectively or potentially?  I don't think consciousness (at least like ours) can occur except in the context of a quasi-classical world.  So it depends on whether the computations are sufficient to instantiate such a world.


 

That we can only access N-bits of a mind from any one world is irrelevant, as all the conscious states exist in the intermediate states,

That's your story and you're sticking to it.


Do you disagree?

It is certainly relevant that we can only access N-bits of an N-qubit computer.  But what it shows is not certain.

Brent

Jason Resch

unread,
Dec 30, 2013, 5:20:52 PM12/30/13
to Everything List
On Mon, Dec 30, 2013 at 4:45 PM, meekerdb <meek...@verizon.net> wrote:
On 12/30/2013 1:29 PM, Jason Resch wrote:



On Mon, Dec 30, 2013 at 3:57 PM, meekerdb <meek...@verizon.net> wrote:
On 12/30/2013 12:04 PM, Jason Resch wrote:



On Mon, Dec 30, 2013 at 2:41 PM, meekerdb <meek...@verizon.net> wrote:
On 12/30/2013 11:17 AM, Jason Resch wrote:



On Mon, Dec 30, 2013 at 2:00 PM, meekerdb <meek...@verizon.net> wrote:
On 12/30/2013 3:09 AM, Bruno Marchal wrote:
But that's essentially everything, since everything is (presumably) quantum.  But notice the limitation of quantum computers, if it has N qubits it takes 2^N complex numbers to specify its state, BUT you can only retrieve N bits of information from it (c.f. Holevo's theorem).  So it doesn't really act like 2^N parallel computers.


OK, but nobody pretended the contrary.  You can still extract N bits depending on the 2^N results, by doing some Fourier transfrom on all results obtained in "parallel universes". This means that the 2^N computations have to occur in *some* sense.

But they pretend that the number 2^N is so large that it cannot exist in whole universe, much less in that little quantum computer and therefore there must be other worlds which contain these enormous number of bits.  What Holevo's theorem shows is the one can regard all those interference terms as mere calculation fictions in going from N bit inputs to N bit outputs. 

Can such "calculation fictions" support conciousness?  That's the real question.  If they can, then you can't avoid many-worlds (or at least many minds).

Why is that "the real question"?  Saying yes to the doctor implies that a classical computer can support consciousness.

Because with computationalism, if a quantum computer runs the computations that support a mind, there would be many resulting conscious states, and first person views. 

Of course that is assuming the very proposition you're arguing.


No, I am trying to show that given computationalism, there is nothing "fictional" about these computations. They would have very bit the same power to yield consciousness as the computations of a classical computer.  Do you disagree with this?

I'm not sure what you mean by "power";

"ability"
 
whether it means effectively or potentially?  I don't think consciousness (at least like ours) can occur except in the context of a quasi-classical world. 

Each of the myriad of computations executed in the quantum computer can be seen as separate classical computations. I agree classical computation is what is behind consciousness, so if quantum computation is the superposition of many classical computations, and if these classical computations instantiate minds, then the emulation of a mind on a quantum computer gives you many different conscious states existing at once.

Our own classical world, is based on the quantum, so really, we don't even need to run a brain simulation in a quantum computer (that is already what is happening to us today, right now).
 
So it depends on whether the computations are sufficient to instantiate such a world.


 

That we can only access N-bits of a mind from any one world is irrelevant, as all the conscious states exist in the intermediate states,

That's your story and you're sticking to it.


Do you disagree?

It is certainly relevant that we can only access N-bits of an N-qubit computer.  But what it shows is not certain.

Brent

--

meekerdb

unread,
Dec 30, 2013, 5:32:30 PM12/30/13
to everyth...@googlegroups.com
On 12/30/2013 2:20 PM, Jason Resch wrote:



On Mon, Dec 30, 2013 at 4:45 PM, meekerdb <meek...@verizon.net> wrote:
On 12/30/2013 1:29 PM, Jason Resch wrote:



On Mon, Dec 30, 2013 at 3:57 PM, meekerdb <meek...@verizon.net> wrote:
On 12/30/2013 12:04 PM, Jason Resch wrote:



On Mon, Dec 30, 2013 at 2:41 PM, meekerdb <meek...@verizon.net> wrote:
On 12/30/2013 11:17 AM, Jason Resch wrote:



On Mon, Dec 30, 2013 at 2:00 PM, meekerdb <meek...@verizon.net> wrote:
On 12/30/2013 3:09 AM, Bruno Marchal wrote:
But that's essentially everything, since everything is (presumably) quantum.  But notice the limitation of quantum computers, if it has N qubits it takes 2^N complex numbers to specify its state, BUT you can only retrieve N bits of information from it (c.f. Holevo's theorem).  So it doesn't really act like 2^N parallel computers.


OK, but nobody pretended the contrary.  You can still extract N bits depending on the 2^N results, by doing some Fourier transfrom on all results obtained in "parallel universes". This means that the 2^N computations have to occur in *some* sense.

But they pretend that the number 2^N is so large that it cannot exist in whole universe, much less in that little quantum computer and therefore there must be other worlds which contain these enormous number of bits.  What Holevo's theorem shows is the one can regard all those interference terms as mere calculation fictions in going from N bit inputs to N bit outputs. 

Can such "calculation fictions" support conciousness?  That's the real question.  If they can, then you can't avoid many-worlds (or at least many minds).

Why is that "the real question"?  Saying yes to the doctor implies that a classical computer can support consciousness.

Because with computationalism, if a quantum computer runs the computations that support a mind, there would be many resulting conscious states, and first person views. 

Of course that is assuming the very proposition you're arguing.


No, I am trying to show that given computationalism, there is nothing "fictional" about these computations. They would have very bit the same power to yield consciousness as the computations of a classical computer.  Do you disagree with this?

I'm not sure what you mean by "power";

"ability"
 
whether it means effectively or potentially?  I don't think consciousness (at least like ours) can occur except in the context of a quasi-classical world. 

Each of the myriad of computations executed in the quantum computer can be seen as separate classical computations. I agree classical computation is what is behind consciousness, so if quantum computation is the superposition of many classical computations,

But that's a very questionable assumption.  If it were literally true then N qubits could do as much a 2^N classical computers, but they can't.  The "quantum computations" are not just classical computations being done in parallel because they have to interfere to produce an answer.

Brent

Jason Resch

unread,
Dec 30, 2013, 5:38:11 PM12/30/13
to Everything List
On Mon, Dec 30, 2013 at 5:32 PM, meekerdb <meek...@verizon.net> wrote:
On 12/30/2013 2:20 PM, Jason Resch wrote:



On Mon, Dec 30, 2013 at 4:45 PM, meekerdb <meek...@verizon.net> wrote:
On 12/30/2013 1:29 PM, Jason Resch wrote:



On Mon, Dec 30, 2013 at 3:57 PM, meekerdb <meek...@verizon.net> wrote:
On 12/30/2013 12:04 PM, Jason Resch wrote:



On Mon, Dec 30, 2013 at 2:41 PM, meekerdb <meek...@verizon.net> wrote:
On 12/30/2013 11:17 AM, Jason Resch wrote:



On Mon, Dec 30, 2013 at 2:00 PM, meekerdb <meek...@verizon.net> wrote:
On 12/30/2013 3:09 AM, Bruno Marchal wrote:
But that's essentially everything, since everything is (presumably) quantum.  But notice the limitation of quantum computers, if it has N qubits it takes 2^N complex numbers to specify its state, BUT you can only retrieve N bits of information from it (c.f. Holevo's theorem).  So it doesn't really act like 2^N parallel computers.


OK, but nobody pretended the contrary.  You can still extract N bits depending on the 2^N results, by doing some Fourier transfrom on all results obtained in "parallel universes". This means that the 2^N computations have to occur in *some* sense.

But they pretend that the number 2^N is so large that it cannot exist in whole universe, much less in that little quantum computer and therefore there must be other worlds which contain these enormous number of bits.  What Holevo's theorem shows is the one can regard all those interference terms as mere calculation fictions in going from N bit inputs to N bit outputs. 

Can such "calculation fictions" support conciousness?  That's the real question.  If they can, then you can't avoid many-worlds (or at least many minds).

Why is that "the real question"?  Saying yes to the doctor implies that a classical computer can support consciousness.

Because with computationalism, if a quantum computer runs the computations that support a mind, there would be many resulting conscious states, and first person views. 

Of course that is assuming the very proposition you're arguing.


No, I am trying to show that given computationalism, there is nothing "fictional" about these computations. They would have very bit the same power to yield consciousness as the computations of a classical computer.  Do you disagree with this?

I'm not sure what you mean by "power";

"ability"
 
whether it means effectively or potentially?  I don't think consciousness (at least like ours) can occur except in the context of a quasi-classical world. 

Each of the myriad of computations executed in the quantum computer can be seen as separate classical computations. I agree classical computation is what is behind consciousness, so if quantum computation is the superposition of many classical computations,

But that's a very questionable assumption.  If it were literally true then N qubits could do as much a 2^N classical computers, but they can't.

It's not questionable, because it is explained by many-worlds.  When you measure any of the particles, you get entangled and see only one of its states (not all the states the particle really carries).
 
  The "quantum computations" are not just classical computations being done in parallel because they have to interfere to produce an answer.

Yes, but I am starting to think you believe quantum computations are more than mere "fictions", since you keep avoiding the question of whether or not multiple conscious states might be supported by a quantum computation.

Jason

Richard Ruquist

unread,
Dec 30, 2013, 7:10:22 PM12/30/13
to everyth...@googlegroups.com
On Mon, Dec 30, 2013 at 4:34 PM, Jason Resch <jason...@gmail.com> wrote:



On Mon, Dec 30, 2013 at 4:00 PM, John Clark <johnk...@gmail.com> wrote:
On Sun, Dec 29, 2013 at 4:52 PM, Jason Resch <jason...@gmail.com> wrote:

>> That means you think things are realistic, and that means  I know for a fact your thinking is wrong, not crazy but wrong. We know from experiment that Bell's inequality is violated, and that means that locality or realism or both MUST be wrong.

> Or measurements are multi-valued.  MWI has both locality and realism.

If the many World's Theory was local AND realistic we'd know with certainty that it's wrong because any theory that is consistent with experimental results can NOT be both. 

There are at least two possible answers to the bell inequalities:
1. Nonlocal influences
2. Mutliple outcomes for each measurement

It appears that string theory (quantum gravity) is chosing 1. The claim of Maldacens and Susskind is that EPR
happens because of tunneling or Einstein Rosen ER bridges resulting in the (bumper sticker or T shirt) ER=EPR.

LizR

unread,
Dec 30, 2013, 7:39:16 PM12/30/13
to everyth...@googlegroups.com
...yrtemmys emit rO

Jesse Mazer

unread,
Dec 30, 2013, 7:51:11 PM12/30/13
to everyth...@googlegroups.com
Even if this connection between entanglement and wormholes holds up, I don't think it automatically means quantum physics is nonlocal and we must discard the many-worlds claim to preserve locality. Keep in mind that in general relativity nothing can actually pass from one end of an Einstein-Rosen bridge to the other, everything that falls in is annihilated in the central singularity, though things falling in from either side can meet in the interior region before they hit the singularity (to get a "traversable" wormhole that actually can be crossed, you need exotic matter to hold it open). But apparently the authors of the paper you cite are suggesting a connection between entanglement and the idea known as "black hole complementarity" which says that two seemingly contradictory claims about what happens to information falling into the black hole (one which says that anything approaching the horizon is destroyed by intense heat and converted to thermal radiation, and another which says it passes smoothly into the interior region) can both be correct from the perspectives of different observers inside and outside the horizon who can never communicate (see https://en.wikipedia.org/wiki/Black_hole_complementarity ). There's a discussion of the connection at http://quantumfrontiers.com/2013/06/07/entanglement-wormholes/ which I don't understand very well, but it does say both "That wormholes are not traversable is important for the consistency of ER = EPR: just as Alice cannot use their shared entanglement to send a message to Bob instantaneously, so she is unable to send Bob a message through their shared wormhole" and also "The ER = EPR conjecture seems to allow us to view the early radiation with which the black hole is entangled as a complementary description of the black hole interior."

Jesse

Bruno Marchal

unread,
Dec 31, 2013, 4:37:37 AM12/31/13
to everyth...@googlegroups.com
On 30 Dec 2013, at 20:00, meekerdb wrote:

On 12/30/2013 3:09 AM, Bruno Marchal wrote:
But that's essentially everything, since everything is (presumably) quantum.  But notice the limitation of quantum computers, if it has N qubits it takes 2^N complex numbers to specify its state, BUT you can only retrieve N bits of information from it (c.f. Holevo's theorem).  So it doesn't really act like 2^N parallel computers.


OK, but nobody pretended the contrary.  You can still extract N bits depending on the 2^N results, by doing some Fourier transfrom on all results obtained in "parallel universes". This means that the 2^N computations have to occur in *some* sense.

But they pretend that the number 2^N is so large that it cannot exist in whole universe, much less in that little quantum computer and therefore there must be other worlds which contain these enormous number of bits.  What Holevo's theorem shows is the one can regard all those interference terms as mere calculation fictions in going from N bit inputs to N bit outputs.  It is conceptually no different than doing a calculation in ordinary probability theory: I start with some initial conditions and I introduce a probability distribution and compute a probability for some event.  In that intermediate step I introduced a continuous probability distribution which implies an *infinite* number of bits.  Nobody thinks this requires an infinite number of worlds.

Then you need to add some selection principle to QM. If QC works through QM, the "parallel" computation are done in our quasi-classical world as in any other branch, and this is tested by doing a Fourier Transform which required the computation do be done in some non fictitious way (or you are adding some non linear magic in QM at some place).


Bruno





Brent


--
You received this message because you are subscribed to the Google Groups "Everything List" group.
To unsubscribe from this group and stop receiving emails from it, send an email to everything-li...@googlegroups.com.
To post to this group, send email to everyth...@googlegroups.com.
Visit this group at http://groups.google.com/group/everything-list.
For more options, visit https://groups.google.com/groups/opt_out.

Bruno Marchal

unread,
Dec 31, 2013, 4:46:00 AM12/31/13
to everyth...@googlegroups.com
On 30 Dec 2013, at 21:49, John Clark wrote:

On Sun, Dec 29, 2013 at 4:09 PM, meekerdb <meek...@verizon.net> wrote:

> If an influence can go backward in time as well as forward then it can effectively have FTL influence,

We already know for a fact that faster than light influences exist, and this has nothing to do with any theory, it was found experimentally.

There is no faster than light influences in QM. You need to have an explicit physical collapse to have that. In the MW, the non locality is only apparent.




Of course this does not mean you can sent a message faster than light, matter and energy and information are still limited by the speed of light, but influences can certainly go astronomically faster, probably infinitely faster.

I suggest you do by yourself a thorough analysis of quantum teleportation in the MW setting. You can see well why it looks like faster than light influence exist, but there are none. Alice eventually just tells Bob in which partition of the multiverse they have situated themselves in.

The existence of "spooky" action at a distance is not supported by QM, only by QM + "one unique cosmos". The splitting of universe (or their differentiation) is a local phenomenon which spread at the speed of physical interaction, which is slower than c.

Bruno



Bruno Marchal

unread,
Dec 31, 2013, 4:51:28 AM12/31/13
to everyth...@googlegroups.com
On 30 Dec 2013, at 22:00, John Clark wrote:

On Sun, Dec 29, 2013 at 4:52 PM, Jason Resch <jason...@gmail.com> wrote:

>> That means you think things are realistic, and that means  I know for a fact your thinking is wrong, not crazy but wrong. We know from experiment that Bell's inequality is violated, and that means that locality or realism or both MUST be wrong.

> Or measurements are multi-valued.  MWI has both locality and realism.

If the many World's Theory was local AND realistic we'd know with certainty that it's wrong because any theory that is consistent with experimental results can NOT be both. 

Only with adding "uniqueness" of the outcomes.



But MWI could be true because although it is realistic it is not local.

I have never seen a proof of this. All proofs assume uniqueness of outcome. I agree that some proof that MW is local admits loophole, but that does not mean that such action at a distance exist, and proof that they exist admits loophole too.



A entire parallel universe as big as our own that you can never go to or even see is about as far from being local as you can get.

Differentiation/splitting of "universes" is a local phenomenon. It is not instantaneous at all.

Bruno







Bruno Marchal

unread,
Dec 31, 2013, 4:57:35 AM12/31/13
to everyth...@googlegroups.com
On 30 Dec 2013, at 23:32, meekerdb wrote:

On 12/30/2013 2:20 PM, Jason Resch wrote:



On Mon, Dec 30, 2013 at 4:45 PM, meekerdb <meek...@verizon.net> wrote:
On 12/30/2013 1:29 PM, Jason Resch wrote:



On Mon, Dec 30, 2013 at 3:57 PM, meekerdb <meek...@verizon.net> wrote:
On 12/30/2013 12:04 PM, Jason Resch wrote:



On Mon, Dec 30, 2013 at 2:41 PM, meekerdb <meek...@verizon.net> wrote:
On 12/30/2013 11:17 AM, Jason Resch wrote:



On Mon, Dec 30, 2013 at 2:00 PM, meekerdb <meek...@verizon.net> wrote:
On 12/30/2013 3:09 AM, Bruno Marchal wrote:
But that's essentially everything, since everything is (presumably) quantum.  But notice the limitation of quantum computers, if it has N qubits it takes 2^N complex numbers to specify its state, BUT you can only retrieve N bits of information from it (c.f. Holevo's theorem).  So it doesn't really act like 2^N parallel computers.


OK, but nobody pretended the contrary.  You can still extract N bits depending on the 2^N results, by doing some Fourier transfrom on all results obtained in "parallel universes". This means that the 2^N computations have to occur in *some* sense.

But they pretend that the number 2^N is so large that it cannot exist in whole universe, much less in that little quantum computer and therefore there must be other worlds which contain these enormous number of bits.  What Holevo's theorem shows is the one can regard all those interference terms as mere calculation fictions in going from N bit inputs to N bit outputs. 

Can such "calculation fictions" support conciousness?  That's the real question.  If they can, then you can't avoid many-worlds (or at least many minds).

Why is that "the real question"?  Saying yes to the doctor implies that a classical computer can support consciousness.

Because with computationalism, if a quantum computer runs the computations that support a mind, there would be many resulting conscious states, and first person views. 

Of course that is assuming the very proposition you're arguing.


No, I am trying to show that given computationalism, there is nothing "fictional" about these computations. They would have very bit the same power to yield consciousness as the computations of a classical computer.  Do you disagree with this?

I'm not sure what you mean by "power";

"ability"
 
whether it means effectively or potentially?  I don't think consciousness (at least like ours) can occur except in the context of a quasi-classical world. 

Each of the myriad of computations executed in the quantum computer can be seen as separate classical computations. I agree classical computation is what is behind consciousness, so if quantum computation is the superposition of many classical computations,

But that's a very questionable assumption.  If it were literally true then N qubits could do as much a 2^N classical computers, but they can't. 

That does not follow. They can't because QM predicts that they can't interact, but the interference needs them to exist, in some physical non fictitious sense. Without adding a selection principle, like a collapse, I don't see why self-aware creature in those branches would lost their consciousness.



The "quantum computations" are not just classical computations being done in parallel because they have to interfere to produce an answer.

So you agree that they are computations in parallel. Then we cannot exploit the results obtained in the parallel world directly, as we would lost the information in the other branch, but by changing the base of the outcome-analyser, we can still exploit some amount of information from the results obtained in *all* other branches (like seeing if they are all equal, or not).

Bruno

John Clark

unread,
Dec 31, 2013, 12:12:26 PM12/31/13
to everyth...@googlegroups.com
On Mon, Dec 30, 2013 at 4:34 PM, Jason Resch <jason...@gmail.com> wrote:

> There are at least two possible answers to the bell inequalities:
1. Nonlocal influences
 
 There are not "at least two" there are exactly two, but yes, things might not be local.

>2. Mutliple outcomes for each measurement

Yes, things might not be realistic. We know that at best one of those 2 commonplace assumptions is wrong, at worse both are.

> If you choose 2, then you don't need 1.

Yes, but locality OR realism OR both must be wrong.

>> But MWI could be true because although it is realistic it is not local.

> It is local,

I sorta like the MWI but apparently you are not a fan because if what you say is true then the MWI is dead wrong. We already know MWI is realistic and ANY theory that is both realistic AND local can NOT be consistent with experiment. And if experiment says that's not the way things are then that's just not the way things are.

> You can have multiple outcomes for a measurement and realism.

No you can not because that's not what physicists mean when they use the word "realistic", they mean that a wave or a particle possesses one specific attribute even if it has not been measured. For example, if a photon already has one specific  polarization even before its quantum entangled twin has been measured then it is realistic.

> Locality has a specific definition in physics,

Yes.

> that things are only affected by other things (fields or particles) in direct proximity to each other.

Once a universe has split off it can have no effect on us whatsoever nor us to it. And someplace that the laws of physics forbid us from going to or seeing is not in our "direct proximity".

> It says nothing about the existence of places we can or can't go to.

It most certainly does! If a event is not even in our past or future spacetime lightcone then it is not local, and no event in another universe is within our lightcone.

  John K Clark

















It says nothing about the existence of places we can or can't go to.

Jason
 

  John K Clark




 





John Clark

unread,
Dec 31, 2013, 12:24:30 PM12/31/13
to everyth...@googlegroups.com
On Tue, Dec 31, 2013 at 4:46 AM, Bruno Marchal <mar...@ulb.ac.be> wrote:

> There is no faster than light influences in QM. You need to have an explicit physical collapse to have that. In the MW, the non locality is only apparent.

So it's all only apparent. I hate it when people say X is a illusion without even hinting at how that illusion works. 

> Alice eventually just tells Bob in which partition of the multiverse they have situated themselves in.

Alice tells Bob? The problem is that the laws of physics provide no way for Alice to say one word to Bob nor Bob to Alice.

  John K Clark



Jason Resch

unread,
Dec 31, 2013, 12:54:42 PM12/31/13
to Everything List
On Tue, Dec 31, 2013 at 12:12 PM, John Clark <johnk...@gmail.com> wrote:
On Mon, Dec 30, 2013 at 4:34 PM, Jason Resch <jason...@gmail.com> wrote:

> There are at least two possible answers to the bell inequalities:
1. Nonlocal influences
 
 There are not "at least two" there are exactly two, but yes, things might not be local.

>2. Mutliple outcomes for each measurement

Yes, things might not be realistic. We know that at best one of those 2 commonplace assumptions is wrong, at worse both are.

> If you choose 2, then you don't need 1.

Yes, but locality OR realism OR both must be wrong.

>> But MWI could be true because although it is realistic it is not local.

> It is local,

I sorta like the MWI but apparently you are not a fan because if what you say is true then the MWI is dead wrong.

Explain why the following table shows that MWI is local, and realistic on the wave function and universal wave function:


Jason
 
We already know MWI is realistic and ANY theory that is both realistic AND local can NOT be consistent with experiment. And if experiment says that's not the way things are then that's just not the way things are.

> You can have multiple outcomes for a measurement and realism.

No you can not because that's not what physicists mean when they use the word "realistic", they mean that a wave or a particle possesses one specific attribute even if it has not been measured.

That is hidden variable. There cannot be a single hidden value but there can be multiple real values.  Hidden variables are something different from realism, which is a reality independent of measurement or observation.

Perhaps that is the only thing we are arguing over.  Definitions.  What you say seems correct to me if what you call reality is things possess "single-valued hidden variables".
 
For example, if a photon already has one specific  polarization even before its quantum entangled twin has been measured then it is realistic.


It has many specific polarizations before it is measured.
 
> Locality has a specific definition in physics,

Yes.

> that things are only affected by other things (fields or particles) in direct proximity to each other.

Once a universe has split off it can have no effect on us whatsoever nor us to it. And someplace that the laws of physics forbid us from going to or seeing is not in our "direct proximity".

The whole universe doesn't split off, rather superpositions spread from particle to particle at sub-light speeds.

 

> It says nothing about the existence of places we can or can't go to.

It most certainly does! If a event is not even in our past or future spacetime lightcone then it is not local, and no event in another universe is within our lightcone.

By this definition, the existence of light cones or things outside would make special relativity non-local, but it is not.  A theory is only non-local if something outside your past light cone could affect you, or if you could affect things outside your future light cone.  This is not the case in special relativity, and it's not the case in Everett's theory.

Jason

Bruno Marchal

unread,
Dec 31, 2013, 1:33:30 PM12/31/13
to everyth...@googlegroups.com
On 31 Dec 2013, at 18:24, John Clark wrote:

On Tue, Dec 31, 2013 at 4:46 AM, Bruno Marchal <mar...@ulb.ac.be> wrote:

> There is no faster than light influences in QM. You need to have an explicit physical collapse to have that. In the MW, the non locality is only apparent.

So it's all only apparent. I hate it when people say X is a illusion without even hinting at how that illusion works. 

See Steve Price FA on Everett, or Tipler's paper, or Deutsch and Hayden paper, or keep in mind the universal wave is "just" a wave evolving without any action at a distance or any indeterminacy in its evolution. Quantum computer are Turing (classicaly) emulable.




> Alice eventually just tells Bob in which partition of the multiverse they have situated themselves in.

Alice tells Bob? The problem is that the laws of physics provide no way for Alice to say one word to Bob nor Bob to Alice.

I was alluding to the moment, in the quantum teleportation protocol, when Alice send classicaly the two supplementary bits of information to Bob so that it can recuperate the information in which partition of the multiverse he is. There is nothing non local.
Those two supplementary bits needs to be send classicaly to "transform" the quantum teleportation (of an arbitrary qubit).

Bruno



  John K Clark




--
You received this message because you are subscribed to the Google Groups "Everything List" group.
To unsubscribe from this group and stop receiving emails from it, send an email to everything-li...@googlegroups.com.
To post to this group, send email to everyth...@googlegroups.com.
Visit this group at http://groups.google.com/group/everything-list.
For more options, visit https://groups.google.com/groups/opt_out.

meekerdb

unread,
Dec 31, 2013, 3:19:56 PM12/31/13
to everyth...@googlegroups.com
On 12/31/2013 1:37 AM, Bruno Marchal wrote:

On 30 Dec 2013, at 20:00, meekerdb wrote:

On 12/30/2013 3:09 AM, Bruno Marchal wrote:
But that's essentially everything, since everything is (presumably) quantum.  But notice the limitation of quantum computers, if it has N qubits it takes 2^N complex numbers to specify its state, BUT you can only retrieve N bits of information from it (c.f. Holevo's theorem).  So it doesn't really act like 2^N parallel computers.


OK, but nobody pretended the contrary.  You can still extract N bits depending on the 2^N results, by doing some Fourier transfrom on all results obtained in "parallel universes". This means that the 2^N computations have to occur in *some* sense.

But they pretend that the number 2^N is so large that it cannot exist in whole universe, much less in that little quantum computer and therefore there must be other worlds which contain these enormous number of bits.  What Holevo's theorem shows is the one can regard all those interference terms as mere calculation fictions in going from N bit inputs to N bit outputs.  It is conceptually no different than doing a calculation in ordinary probability theory: I start with some initial conditions and I introduce a probability distribution and compute a probability for some event.  In that intermediate step I introduced a continuous probability distribution which implies an *infinite* number of bits.  Nobody thinks this requires an infinite number of worlds.

Then you need to add some selection principle to QM. If QC works through QM, the "parallel" computation are done in our quasi-classical world as in any other branch, and this is tested by doing a Fourier Transform which required the computation do be done in some non fictitious way (or you are adding some non linear magic in QM at some place).

I don't understand your comment.  It is my point that the computations are done in our world via the interference of wave functions - which have to be in the same world in order to interfere.  I take "worlds" to mean quasi-classical worlds and a quasi-classical world may be supported by many different quantum "worlds".  But in Deutsch's famous proposed experiment, a quantum AI computer after factoring some prime by Shor's algorithm may not be able to tell us anything more than the factors - because those factors comes out the same in almost all branches and hence correspond to the same quasi-classical world.

Brent

meekerdb

unread,
Dec 31, 2013, 3:52:59 PM12/31/13
to everyth...@googlegroups.com
On 12/31/2013 1:51 AM, Bruno Marchal wrote:
A entire parallel universe as big as our own that you can never go to or even see is about as far from being local as you can get.

Differentiation/splitting of "universes" is a local phenomenon. It is not instantaneous at all.

In an EPR experiment if you imagine a forward light cone of entanglement issuing from Alice and another from Bob those are STL effects (aka local), but where the light cones start to overlap it is observed that there is more correlation than can be accounted for by a local variables at the EPR source.  That's sort of the definition of "influence".  So the split into different quasi-classical worlds didn't just start from Alice or from Bob, they had to split in a a correlated way.

Brent

meekerdb

unread,
Dec 31, 2013, 4:11:11 PM12/31/13
to everyth...@googlegroups.com
That's pretty funny coming from you, Bruno.  :-)


Without adding a selection principle, like a collapse, I don't see why self-aware creature in those branches would lost their consciousness.

The question is why are "they" not us.  We remain self-aware while the quantum computer factors a number, so we're self-aware creatures in the same branch.





The "quantum computations" are not just classical computations being done in parallel because they have to interfere to produce an answer.

So you agree that they are computations in parallel.

Sure. I'm just doubtful they constitute "a world".


Then we cannot exploit the results obtained in the parallel world directly, as we would lost the information in the other branch, but by changing the base of the outcome-analyser, we can still exploit some amount of information from the results obtained in *all* other branches (like seeing if they are all equal, or not).

Effectively that's how most quantum computations work, they don't guarantee the right answer, only a statistically probable answer.  So we're relying on the agreement of a lot of parallel processes to interfere constructively on the right answer.

Brent

John Clark

unread,
Dec 31, 2013, 4:24:16 PM12/31/13
to everyth...@googlegroups.com
On Tue, Dec 31, 2013 at 12:54 PM, Jason Resch <jason...@gmail.com> wrote:

>> I sorta like the MWI but apparently you are not a fan because if what you say is true then the MWI is dead wrong.

> Explain why the following table shows that MWI is local, and realistic on the wave function and universal wave function:

I have no idea where Wikipedia got that table but it is self contradictory. It says the observer plays no part in many worlds but it also says "no" to counterfactual definiteness meaning you can't speak meaningfully of the definiteness of the results of measurements that have not been observed. Both those things can't be right. And in many world there is no unique future but it says there is no unique past, and that's not what the theory says.

>>that's not what physicists mean when they use the word "realistic", they mean that a wave or a particle possesses one specific attribute even if it has not been measured.

> That is hidden variable.

That is realism. Hidden variables are about how something is going to change, realism is about how something is right now.

> There cannot be a single hidden value but there can be multiple real values.

I don't know what you mean by that. The best way to think of hidden variables is as a lookup table that photons and electrons can see but for some reason we can not. The table can contain as many real values as you like, it can even contain an infinite number of values; but no lookup table, no set of hidden variables, can explain the results we see from experiment. 

>>For example, if a photon already has one specific  polarization even before its quantum entangled twin has been measured then it is realistic.

> It has many specific polarizations before it is measured.

Then it is observer dependent, and the crown jewel of the MWI is that it's observer independent and solves the measurement problem. The MWI may have other difficulties but at least it solves the measurement problem, without that WMI has no advantage and you might as well stick with the Copenhagen muddle.

It says "the splitting is a local process, transmitted causally at light or sub-light speeds", so the question is, transmitted THROUGH WHAT at light or sub-light speeds? I don't know but it certainly isn't through local space.

>>> It says nothing about the existence of places we can or can't go to.

>> It most certainly does! If a event is not even in our past or future spacetime lightcone then it is not local, and no event in another universe is within our lightcone.

> By this definition, the existence of light cones or things outside would make special relativity non-local,

No it does not because non-local events, that is to say things  outside our past or future lightcone may exist but they have no effect on what we see here and now nor can anything that happens here effect anything there.

 > A theory is only non-local if something outside your past light cone could affect you, or if you could affect things outside your future light cone.

Exactly.
 
  > This is not the case in special relativity,

Exactly. So how is it non-local?

  John K Clark





Jason Resch

unread,
Dec 31, 2013, 5:07:50 PM12/31/13
to Everything List
On Tue, Dec 31, 2013 at 4:24 PM, John Clark <johnk...@gmail.com> wrote:
On Tue, Dec 31, 2013 at 12:54 PM, Jason Resch <jason...@gmail.com> wrote:

>> I sorta like the MWI but apparently you are not a fan because if what you say is true then the MWI is dead wrong.

> Explain why the following table shows that MWI is local, and realistic on the wave function and universal wave function:

I have no idea where Wikipedia got that table but it is self contradictory. It says the observer plays no part in many worlds

I would add a (*) on "observer role". In MWI the observer plays no special function in the evolution of the wave function. This is not the case for many interpretations where the observer plays some special privileged role, such as having the ability to collapse wave functions.
 
but it also says "no" to counterfactual definiteness meaning you can't speak meaningfully of the definiteness of the results of measurements that have not been observed.

That is true for MWI because measurements don't have (single) definite results.
 
Both those things can't be right.

Can you explain why not?
 
And in many world there is no unique future but it says there is no unique past, and that's not what the theory says.

There is no unique past as shown in the quantum erasure experiment.
 

>>that's not what physicists mean when they use the word "realistic", they mean that a wave or a particle possesses one specific attribute even if it has not been measured.

> That is hidden variable.

That is realism. Hidden variables are about how something is going to change, realism is about how something is right now.

The wave function says everything there is to be said about how something is right now. The MWI says this wave function is real.
 

> There cannot be a single hidden value but there can be multiple real values.

I don't know what you mean by that. The best way to think of hidden variables is as a lookup table that photons and electrons can see but for some reason we can not. The table can contain as many real values as you like, it can even contain an infinite number of values; but no lookup table, no set of hidden variables, can explain the results we see from experiment.

It can. If a particle has multiple values (spin up and spin down) and those values correspond to another far away particle, which also has two values (spin down and spin up), and if when someone measures that particle, they too become correlated with the dual states of the particle, this explains the Bell inequalities.  It is only when one presumes a measurement can have only a single definite value that you need FTL influences to explain the Bell inequalities. The assumption that measurements could only have one outcome was "so obvious" Bell (and many others) didn't even realize they had made this assumption.
 
 

>>For example, if a photon already has one specific  polarization even before its quantum entangled twin has been measured then it is realistic.

> It has many specific polarizations before it is measured.

Then it is observer dependent, and the crown jewel of the MWI is that it's observer independent and solves the measurement problem.

It does, and MWI perfectly explains how measurement works. The multiple values are not created or set by the measurement.  Consider a photon that has passed through a half-silvered mirror, such that it has two velocities and two positions. This photon's position and momentum have two values.  Now imagine in only one of the directions the photon is traversing, it is heading straight toward an electron.  Now when this photon (with its two directions) interacts with the electron, it both hits and misses the electron. The electron that is hit is correlated with the photon that traveled the path toward it, and it is missed which is correlated with the photon that took the alternate path.  Thus, the "multi-positioned" photon, upon interacting and not interacting with this electron, has put the electron into two states (of being deflected and not being deflected), and the correlations are explained.  Now imagine the deflected photon hits a TV screen causing it to flash.  The electron which is deflected and not deflected, may interact and not interact with the screen, and photons emitted from the screen and not emitted from the screen may cause an observer to see and not see the flash.  Now all the positions of all the particles in the person's  brain are in different positions, because they remember seeing a flash in one "branch", and remember seeing no flash in the other. These correlations and multiplications of states spread from particle to particle, whether or not the particle is a free electron or a light-sensing rod in a conscious observer's retina.
 
The MWI may have other difficulties but at least it solves the measurement problem, without that WMI has no advantage and you might as well stick with the Copenhagen muddle.

It says "the splitting is a local process, transmitted causally at light or sub-light speeds", so the question is, transmitted THROUGH WHAT at light or sub-light speeds? I don't know but it certainly isn't through local space.

Transmitted through the background space these "multi-valued" particles exist in.
 


>>> It says nothing about the existence of places we can or can't go to.

>> It most certainly does! If a event is not even in our past or future spacetime lightcone then it is not local, and no event in another universe is within our lightcone.

> By this definition, the existence of light cones or things outside would make special relativity non-local,

No it does not because non-local events, that is to say things  outside our past or future lightcone may exist but they have no effect on what we see here and now nor can anything that happens here effect anything there.

 > A theory is only non-local if something outside your past light cone could affect you, or if you could affect things outside your future light cone.

Exactly.

So when, in the context of MWI, does this ever happen?

Jason
 
 
  > This is not the case in special relativity,

Exactly. So how is it non-local?

  John K Clark





meekerdb

unread,
Dec 31, 2013, 6:05:29 PM12/31/13
to everyth...@googlegroups.com
On 12/31/2013 9:54 AM, Jason Resch wrote:



On Tue, Dec 31, 2013 at 12:12 PM, John Clark <johnk...@gmail.com> wrote:
On Mon, Dec 30, 2013 at 4:34 PM, Jason Resch <jason...@gmail.com> wrote:

> There are at least two possible answers to the bell inequalities:
1. Nonlocal influences
 
 There are not "at least two" there are exactly two, but yes, things might not be local.

>2. Mutliple outcomes for each measurement

Yes, things might not be realistic. We know that at best one of those 2 commonplace assumptions is wrong, at worse both are.

> If you choose 2, then you don't need 1.

Yes, but locality OR realism OR both must be wrong.

>> But MWI could be true because although it is realistic it is not local.

> It is local,

I sorta like the MWI but apparently you are not a fan because if what you say is true then the MWI is dead wrong.

Explain why the following table shows that MWI is local, and realistic on the wave function and universal wave function:



The Wikipedia reference:

Locality in the Everett Interpretation of Heisenberg-Picture Quantum Mechanics

(Submitted on 14 Mar 2001 (v1), last revised 10 May 2001 (this version, v2))
Bell's theorem depends crucially on counterfactual reasoning, and is mistakenly interpreted as ruling out a local explanation for the correlations which can be observed between the results of measurements performed on spatially-separated quantum systems. But in fact the Everett interpretation of quantum mechanics, in the Heisenberg picture, provides an alternative local explanation for such correlations. Measurement-type interactions lead, not to many worlds but, rather, to many local copies of experimental systems and the observers who measure their properties. Transformations of the Heisenberg-picture operators corresponding to the properties of these systems and observers, induced by measurement interactions, "label" each copy and provide the mechanism which, e.g., ensures that each copy of one of the observers in an EPRB or GHZM experiment will only interact with the "correct" copy of the other observer(s). The conceptual problem of nonlocality is thus replaced with a conceptual problem of proliferating labels, as correlated systems and observers undergo measurement-type interactions with newly-encountered objects and instruments; it is suggested that this problem may be resolved by considering quantum field theory rather than the quantum mechanics of particles.
Comments: 18 pages, no figures. Minor changes
Subjects: Quantum Physics (quant-ph)
Journal reference: Found. Phys. Lett. 14 (2001) 301-322
Report number: WW-10184
Cite as: arXiv:quant-ph/0103079

just moves the problem from FTL signaling to FTL labeling.



Jason
 
We already know MWI is realistic and ANY theory that is both realistic AND local can NOT be consistent with experiment. And if experiment says that's not the way things are then that's just not the way things are.

> You can have multiple outcomes for a measurement and realism.

No you can not because that's not what physicists mean when they use the word "realistic", they mean that a wave or a particle possesses one specific attribute even if it has not been measured.

That is hidden variable. There cannot be a single hidden value but there can be multiple real values.  Hidden variables are something different from realism, which is a reality independent of measurement or observation.

Perhaps that is the only thing we are arguing over.  Definitions.  What you say seems correct to me if what you call reality is things possess "single-valued hidden variables".
 
For example, if a photon already has one specific  polarization even before its quantum entangled twin has been measured then it is realistic.


It has many specific polarizations before it is measured.
 
> Locality has a specific definition in physics,

Yes.

> that things are only affected by other things (fields or particles) in direct proximity to each other.

Once a universe has split off it can have no effect on us whatsoever nor us to it. And someplace that the laws of physics forbid us from going to or seeing is not in our "direct proximity".

The whole universe doesn't split off, rather superpositions spread from particle to particle at sub-light speeds.


This is a non-answer.  The wave-equation lives in Hilbert space, not spacetime.  So a single point in the Hilbert space of a pair of particles has six space coordinate variables.

Brent

LizR

unread,
Dec 31, 2013, 6:24:03 PM12/31/13
to everyth...@googlegroups.com
On 1 January 2014 12:05, meekerdb <meek...@verizon.net> wrote:
(Submitted on 14 Mar 2001 (v1), last revised 10 May 2001 (this version, v2))
Bell's theorem depends crucially on counterfactual reasoning, and is mistakenly interpreted as ruling out a local explanation for the correlations which can be observed between the results of measurements performed on spatially-separated quantum systems. But in fact the Everett interpretation of quantum mechanics, in the Heisenberg picture, provides an alternative local explanation for such correlations. Measurement-type interactions lead, not to many worlds but, rather, to many local copies of experimental systems and the observers who measure their properties. Transformations of the Heisenberg-picture operators corresponding to the properties of these systems and observers, induced by measurement interactions, "label" each copy and provide the mechanism which, e.g., ensures that each copy of one of the observers in an EPRB or GHZM experiment will only interact with the "correct" copy of the other observer(s). The conceptual problem of nonlocality is thus replaced with a conceptual problem of proliferating labels, as correlated systems and observers undergo measurement-type interactions with newly-encountered objects and instruments; it is suggested that this problem may be resolved by considering quantum field theory rather than the quantum mechanics of particles.
Comments: 18 pages, no figures. Minor changes
Subjects: Quantum Physics (quant-ph)
Journal reference: Found. Phys. Lett. 14 (2001) 301-322
Report number: WW-10184
Cite as: arXiv:quant-ph/0103079

just moves the problem from FTL signaling to FTL labeling.

Where is the FTL? I don't recall any suggestion that the "contagion" of entangled systems spreading themeselves in the MWI involves anything FTL. In fact, it's generally assumed to be very, very STL (unless light itself is involved). At great distances from the laboratory, one imagines that the superposition caused by whatever we might do to cats in boxes would decay to the level of noise, and fail to spread any further. So a gaxlaxy (say) might be in a MWI bubble of superpositions that fails to split neighbouring galaxies for billions of years, because the difference between them is undetectable. Or maybe even planets.... What difference would it make to anyone in M31 if the Nazis had won WW2? (after the light travel time had elapsed, I mean. Maybe a few different radio signals could be picked up, if anyone pointed an antenna in the right direction...)

meekerdb

unread,
Dec 31, 2013, 7:54:21 PM12/31/13
to everyth...@googlegroups.com
On 12/31/2013 3:24 PM, LizR wrote:
On 1 January 2014 12:05, meekerdb <meek...@verizon.net> wrote:
(Submitted on 14 Mar 2001 (v1), last revised 10 May 2001 (this version, v2))
Bell's theorem depends crucially on counterfactual reasoning, and is mistakenly interpreted as ruling out a local explanation for the correlations which can be observed between the results of measurements performed on spatially-separated quantum systems. But in fact the Everett interpretation of quantum mechanics, in the Heisenberg picture, provides an alternative local explanation for such correlations. Measurement-type interactions lead, not to many worlds but, rather, to many local copies of experimental systems and the observers who measure their properties. Transformations of the Heisenberg-picture operators corresponding to the properties of these systems and observers, induced by measurement interactions, "label" each copy and provide the mechanism which, e.g., ensures that each copy of one of the observers in an EPRB or GHZM experiment will only interact with the "correct" copy of the other observer(s). The conceptual problem of nonlocality is thus replaced with a conceptual problem of proliferating labels, as correlated systems and observers undergo measurement-type interactions with newly-encountered objects and instruments; it is suggested that this problem may be resolved by considering quantum field theory rather than the quantum mechanics of particles.
Comments: 18 pages, no figures. Minor changes
Subjects: Quantum Physics (quant-ph)
Journal reference: Found. Phys. Lett. 14 (2001) 301-322
Report number: WW-10184
Cite as: arXiv:quant-ph/0103079

just moves the problem from FTL signaling to FTL labeling.

Where is the FTL? I don't recall any suggestion that the "contagion" of entangled systems spreading themeselves in the MWI involves anything FTL.

Of course in Hilbert space there's no FTL because the system is just one point and when a measurement is performed it projects the system ray onto a mixture of subspaces; spacetime coordinates are just some labels.


In fact, it's generally assumed to be very, very STL (unless light itself is involved). At great distances from the laboratory, one imagines that the superposition caused by whatever we might do to cats in boxes would decay to the level of noise, and fail to spread any further.

That's an interesting viewpoint - but it's taking spacetime instead of Hilbert space to be the arena.  If we take the cat, either alive or dead, and shoot it off into space then, as a signal, it won't fall off as 1/r^2.

Brent

So a gaxlaxy (say) might be in a MWI bubble of superpositions that fails to split neighbouring galaxies for billions of years, because the difference between them is undetectable. Or maybe even planets.... What difference would it make to anyone in M31 if the Nazis had won WW2? (after the light travel time had elapsed, I mean. Maybe a few different radio signals could be picked up, if anyone pointed an antenna in the right direction...)

LizR

unread,
Dec 31, 2013, 10:22:02 PM12/31/13
to everyth...@googlegroups.com
On 1 January 2014 13:54, meekerdb <meek...@verizon.net> wrote:
On 12/31/2013 3:24 PM, LizR wrote:
On 1 January 2014 12:05, meekerdb <meek...@verizon.net> wrote:
(Submitted on 14 Mar 2001 (v1), last revised 10 May 2001 (this version, v2))
Bell's theorem depends crucially on counterfactual reasoning, and is mistakenly interpreted as ruling out a local explanation for the correlations which can be observed between the results of measurements performed on spatially-separated quantum systems. But in fact the Everett interpretation of quantum mechanics, in the Heisenberg picture, provides an alternative local explanation for such correlations. Measurement-type interactions lead, not to many worlds but, rather, to many local copies of experimental systems and the observers who measure their properties. Transformations of the Heisenberg-picture operators corresponding to the properties of these systems and observers, induced by measurement interactions, "label" each copy and provide the mechanism which, e.g., ensures that each copy of one of the observers in an EPRB or GHZM experiment will only interact with the "correct" copy of the other observer(s). The conceptual problem of nonlocality is thus replaced with a conceptual problem of proliferating labels, as correlated systems and observers undergo measurement-type interactions with newly-encountered objects and instruments; it is suggested that this problem may be resolved by considering quantum field theory rather than the quantum mechanics of particles.
Comments: 18 pages, no figures. Minor changes
Subjects: Quantum Physics (quant-ph)
Journal reference: Found. Phys. Lett. 14 (2001) 301-322
Report number: WW-10184
Cite as: arXiv:quant-ph/0103079

just moves the problem from FTL signaling to FTL labeling.

Where is the FTL? I don't recall any suggestion that the "contagion" of entangled systems spreading themeselves in the MWI involves anything FTL.

Of course in Hilbert space there's no FTL because the system is just one point and when a measurement is performed it projects the system ray onto a mixture of subspaces; spacetime coordinates are just some labels.

I thought there was no FTL in ordinary space, either? (I mean, none required for the MWI?)
In fact, it's generally assumed to be very, very STL (unless light itself is involved). At great distances from the laboratory, one imagines that the superposition caused by whatever we might do to cats in boxes would decay to the level of noise, and fail to spread any further.
That's an interesting viewpoint - but it's taking spacetime instead of Hilbert space to be the arena.  If we take the cat, either alive or dead, and shoot it off into space then, as a signal, it won't fall off as 1/r^2.

No, but it will travel STL!

I have the feeling I'm missing the point. Please be gentle with me. 

meekerdb

unread,
Jan 1, 2014, 3:34:31 AM1/1/14
to everyth...@googlegroups.com
On 12/31/2013 7:22 PM, LizR wrote:
On 1 January 2014 13:54, meekerdb <meek...@verizon.net> wrote:
On 12/31/2013 3:24 PM, LizR wrote:
On 1 January 2014 12:05, meekerdb <meek...@verizon.net> wrote:
(Submitted on 14 Mar 2001 (v1), last revised 10 May 2001 (this version, v2))
Bell's theorem depends crucially on counterfactual reasoning, and is mistakenly interpreted as ruling out a local explanation for the correlations which can be observed between the results of measurements performed on spatially-separated quantum systems. But in fact the Everett interpretation of quantum mechanics, in the Heisenberg picture, provides an alternative local explanation for such correlations. Measurement-type interactions lead, not to many worlds but, rather, to many local copies of experimental systems and the observers who measure their properties. Transformations of the Heisenberg-picture operators corresponding to the properties of these systems and observers, induced by measurement interactions, "label" each copy and provide the mechanism which, e.g., ensures that each copy of one of the observers in an EPRB or GHZM experiment will only interact with the "correct" copy of the other observer(s). The conceptual problem of nonlocality is thus replaced with a conceptual problem of proliferating labels, as correlated systems and observers undergo measurement-type interactions with newly-encountered objects and instruments; it is suggested that this problem may be resolved by considering quantum field theory rather than the quantum mechanics of particles.
Comments: 18 pages, no figures. Minor changes
Subjects: Quantum Physics (quant-ph)
Journal reference: Found. Phys. Lett. 14 (2001) 301-322
Report number: WW-10184
Cite as: arXiv:quant-ph/0103079

just moves the problem from FTL signaling to FTL labeling.

Where is the FTL? I don't recall any suggestion that the "contagion" of entangled systems spreading themeselves in the MWI involves anything FTL.

Of course in Hilbert space there's no FTL because the system is just one point and when a measurement is performed it projects the system ray onto a mixture of subspaces; spacetime coordinates are just some labels.

I thought there was no FTL in ordinary space, either? (I mean, none required for the MWI?)

Right, but the state in Hilbert space is something like |x1 y1 z1 s1 x2 y2 z2 s2> and when Alice measures s1 at (x1 y1 z1) then s2 is correlated at (x2 y2 z2).  As I understand it the MWI advocates say this isn't FTL because this is just selecting out one of infinitely many results |s1 s2>.  But the 'selection' has to pair up the spins in a way that violates Bell's inequality.


In fact, it's generally assumed to be very, very STL (unless light itself is involved). At great distances from the laboratory, one imagines that the superposition caused by whatever we might do to cats in boxes would decay to the level of noise, and fail to spread any further.
That's an interesting viewpoint - but it's taking spacetime instead of Hilbert space to be the arena.  If we take the cat, either alive or dead, and shoot it off into space then, as a signal, it won't fall off as 1/r^2.

No, but it will travel STL!

Sure.  I was just commenting on the idea that the entanglement has a kind of limited range because of 'background noise'.  An interesting idea, similar to one I've had that there is a smallest non-zero probability.

But if you want to get FTL, that's possible if Alice and Bob are near opposite sides of our Hubble sphere when they do their measurements.  They are then already moving apart faster than c and will never be able to communicate - with each other, but we, in the middle will eventually receive reports from them so that we can confirm the violation of Bell's inequality.

Brent

LizR

unread,
Jan 1, 2014, 4:33:46 AM1/1/14
to everyth...@googlegroups.com
On 1 January 2014 21:34, meekerdb <meek...@verizon.net> wrote:
On 12/31/2013 7:22 PM, LizR wrote:
On 1 January 2014 13:54, meekerdb <meek...@verizon.net> wrote:
Of course in Hilbert space there's no FTL because the system is just one point and when a measurement is performed it projects the system ray onto a mixture of subspaces; spacetime coordinates are just some labels.

I thought there was no FTL in ordinary space, either? (I mean, none required for the MWI?)
Right, but the state in Hilbert space is something like |x1 y1 z1 s1 x2 y2 z2 s2> and when Alice measures s1 at (x1 y1 z1) then s2 is correlated at (x2 y2 z2).  As I understand it the MWI advocates say this isn't FTL because this is just selecting out one of infinitely many results |s1 s2>.  But the 'selection' has to pair up the spins in a way that violates Bell's inequality.

If I understand correctly ... actually, let me just check if I do, before I go any further, in case I'm talking out my arse. Which wouldn't be the first time.

I assume we're talking about an EPR correlation here?

If yes, I've never understood how the MWI explains this. I've see it explained with ASCII diagrams by Bill Taylor on the FOAR forum, and far be it from me to quibble with Bill, but it never made sense to me. Somehow, the various branches just join up correctly...

The only explanation I've come across that I really understand for EPR, and that doesn't violate locality etc is the time symmetry one, where all influences travel along the light cone, but are allowed to go either way in time.

So although I quite like the MWI because of its ontological implications, this is one point on which I am agnostic, because I don't understand the explanation.


In fact, it's generally assumed to be very, very STL (unless light itself is involved). At great distances from the laboratory, one imagines that the superposition caused by whatever we might do to cats in boxes would decay to the level of noise, and fail to spread any further.
That's an interesting viewpoint - but it's taking spacetime instead of Hilbert space to be the arena.  If we take the cat, either alive or dead, and shoot it off into space then, as a signal, it won't fall off as 1/r^2.

No, but it will travel STL!

Sure.  I was just commenting on the idea that the entanglement has a kind of limited range because of 'background noise'.  An interesting idea, similar to one I've had that there is a smallest non-zero probability.

But if you want to get FTL, that's possible if Alice and Bob are near opposite sides of our Hubble sphere when they do their measurements.  They are then already moving apart faster than c and will never be able to communicate - with each other, but we, in the middle will eventually receive reports from them so that we can confirm the violation of Bell's inequality.

Hmm, that's a good point. That would, however, fit in nicely with time symmetry (which really needs a nice acronym, I'm not sure "TS" cuts it). I tend to evangelise a bit on time symmetry, but only because everyone else roundly ignores it, and it seems to me that it at least has potential.
 

Bruno Marchal

unread,
Jan 1, 2014, 6:01:49 AM1/1/14
to everyth...@googlegroups.com
On 31 Dec 2013, at 21:19, meekerdb wrote:

On 12/31/2013 1:37 AM, Bruno Marchal wrote:

On 30 Dec 2013, at 20:00, meekerdb wrote:

On 12/30/2013 3:09 AM, Bruno Marchal wrote:
But that's essentially everything, since everything is (presumably) quantum.  But notice the limitation of quantum computers, if it has N qubits it takes 2^N complex numbers to specify its state, BUT you can only retrieve N bits of information from it (c.f. Holevo's theorem).  So it doesn't really act like 2^N parallel computers.


OK, but nobody pretended the contrary.  You can still extract N bits depending on the 2^N results, by doing some Fourier transfrom on all results obtained in "parallel universes". This means that the 2^N computations have to occur in *some* sense.

But they pretend that the number 2^N is so large that it cannot exist in whole universe, much less in that little quantum computer and therefore there must be other worlds which contain these enormous number of bits.  What Holevo's theorem shows is the one can regard all those interference terms as mere calculation fictions in going from N bit inputs to N bit outputs.  It is conceptually no different than doing a calculation in ordinary probability theory: I start with some initial conditions and I introduce a probability distribution and compute a probability for some event.  In that intermediate step I introduced a continuous probability distribution which implies an *infinite* number of bits.  Nobody thinks this requires an infinite number of worlds.

Then you need to add some selection principle to QM. If QC works through QM, the "parallel" computation are done in our quasi-classical world as in any other branch, and this is tested by doing a Fourier Transform which required the computation do be done in some non fictitious way (or you are adding some non linear magic in QM at some place).

I don't understand your comment.  It is my point that the computations are done in our world via the interference of wave functions - which have to be in the same world in order to interfere. 

Not sure that this makes sense to me. The wave is the"wave of universe", it evolves in a Hilbert Space, not in any world or branch. The term "world" is confusing here.



I take "worlds" to mean quasi-classical worlds and a quasi-classical world may be supported by many different quantum "worlds".  But in Deutsch's famous proposed experiment, a quantum AI computer after factoring some prime by Shor's algorithm may not be able to tell us anything more than the factors - because those factors comes out the same in almost all branches and hence correspond to the same quasi-classical world.


Because we manage to get the result in our quasi-classical world. Yet, we get an indirect information about what has been computed in the other branch. We can compute a predicate, which outputs 1 or 0, and decide, in our branch, to test if all output were identical or not.

Also with Deutsch quantum brain, the observer can know that he has seen a definite result in each branch, but can't remember which one. So he can test the presence of the alternate realities, but not much more (but still more) than with the usual two slits.

Then in arithmetic, the parallel realities existence is an elementary theorem of RA. So we expect the MW, and are happy that some physicists already confirm this when abandoning the collapse,  ...which is all absurd by itself, note.

Bruno




Bruno Marchal

unread,
Jan 1, 2014, 6:10:56 AM1/1/14
to everyth...@googlegroups.com
On 31 Dec 2013, at 21:52, meekerdb wrote:

On 12/31/2013 1:51 AM, Bruno Marchal wrote:
A entire parallel universe as big as our own that you can never go to or even see is about as far from being local as you can get.

Differentiation/splitting of "universes" is a local phenomenon. It is not instantaneous at all.

In an EPR experiment if you imagine a forward light cone of entanglement issuing from Alice and another from Bob those are STL effects (aka local), but where the light cones start to overlap it is observed that there is more correlation than can be accounted for by a local variables at the EPR source. 

In *one* world, not in the multiverse. See the not to bad explanation in well Everett original papers, or in Steve Price Everett FAQ. 




That's sort of the definition of "influence". 

I don't see any FTL influence.



So the split into different quasi-classical worlds didn't just start from Alice or from Bob, they had to split in a a correlated way.

I don't think so at all. We don't need any FTL influence in each branches of any wave, written in any base, describing the quantum teleportation.
An FTL is needed if we believe that some branch can disappear. If no branch disappear, we have the right partition, and in each (4) branches, the two supplementary classical bits that Alice send classically only informed Bob about his place in the multiverse.

Bruno




It is loading more messages.
0 new messages